You are on page 1of 124

The Physics GRE Solution Guide

GR8677 Test

FT
RA
http://groups.yahoo.com/group/physicsgre_v2

April 15, 2009


D

Author:
David S. Latchman
2

FT
RA
D

David S. Latchman ©2009


Chapter 1

Preface

This solution guide initially started out on the Yahoo Groups web site and was pretty

FT
successful at the time. Unfortunately, the group was lost and with it, much of the the
hard work that was put into it. This is my attempt to recreate the solution guide and
make it more widely avaialble to everyone. If you see any errors, think certain things
could be expressed more clearly, or would like to make suggestions, please feel free to
do so.
David Latchman

Document Changes
RA
04-15-2009 First Version
D
4 Preface

FT
RA
D

David S. Latchman ©2009


Contents

1 Preface 3

2 Classical Mechanics 13
2.1 Kinematics . . . . . . . . . . . . . . . . . . . . . . . . . . . . . . . . . . . . 13

FT
2.1.1 Linear Motion . . . . . . . . . . . . . . . . . . . . . . . . . . . . . . 13
2.1.2 Circular Motion . . . . . . . . . . . . . . . . . . . . . . . . . . . . . 13
2.2 Newton’s Laws . . . . . . . . . . . . . . . . . . . . . . . . . . . . . . . . . 14
2.2.1 Newton’s Laws of Motion . . . . . . . . . . . . . . . . . . . . . . . 14
2.2.2 Momentum . . . . . . . . . . . . . . . . . . . . . . . . . . . . . . . 14
2.2.3 Impulse . . . . . . . . . . . . . . . . . . . . . . . . . . . . . . . . . 14
2.3 Work & Energy . . . . . . . . . . . . . . . . . . . . . . . . . . . . . . . . . 15
2.3.1 Kinetic Energy . . . . . . . . . . . . . . . . . . . . . . . . . . . . . 15
RA
2.3.2 The Work-Energy Theorem . . . . . . . . . . . . . . . . . . . . . . 15
2.3.3 Work done under a constant Force . . . . . . . . . . . . . . . . . . 15
2.3.4 Potential Energy . . . . . . . . . . . . . . . . . . . . . . . . . . . . 15
2.3.5 Hooke’s Law . . . . . . . . . . . . . . . . . . . . . . . . . . . . . . 15
2.3.6 Potential Energy of a Spring . . . . . . . . . . . . . . . . . . . . . . 15
2.4 Oscillatory Motion . . . . . . . . . . . . . . . . . . . . . . . . . . . . . . . 16
2.4.1 Equation for Simple Harmonic Motion . . . . . . . . . . . . . . . 16
2.4.2 Period of Simple Harmonic Motion . . . . . . . . . . . . . . . . . 16
2.4.3 Total Energy of an Oscillating System . . . . . . . . . . . . . . . . 16
2.4.4 Damped Harmonic Motion . . . . . . . . . . . . . . . . . . . . . . 16
D

2.4.5 Small Oscillations . . . . . . . . . . . . . . . . . . . . . . . . . . . . 17


2.4.6 Coupled Harmonic Oscillators . . . . . . . . . . . . . . . . . . . . 17
2.4.7 Doppler Effect . . . . . . . . . . . . . . . . . . . . . . . . . . . . . . 19
2.5 Rotational Motion about a Fixed Axis . . . . . . . . . . . . . . . . . . . . 20
2.5.1 Moment of Inertia . . . . . . . . . . . . . . . . . . . . . . . . . . . 20
2.5.2 Rotational Kinetic Energy . . . . . . . . . . . . . . . . . . . . . . . 20
2.5.3 Parallel Axis Theorem . . . . . . . . . . . . . . . . . . . . . . . . . 20
2.5.4 Torque . . . . . . . . . . . . . . . . . . . . . . . . . . . . . . . . . . 20
2.5.5 Angular Momentum . . . . . . . . . . . . . . . . . . . . . . . . . . 20
2.5.6 Kinetic Energy in Rolling . . . . . . . . . . . . . . . . . . . . . . . 21
2.6 Dynamics of Systems of Particles . . . . . . . . . . . . . . . . . . . . . . . 21
2.6.1 Center of Mass of a System of Particles . . . . . . . . . . . . . . . 21
2.7 Central Forces and Celestial Mechanics . . . . . . . . . . . . . . . . . . . 21
2.7.1 Newton’s Law of Universal Gravitation . . . . . . . . . . . . . . . 21
6 Contents
2.7.2 Potential Energy of a Gravitational Force . . . . . . . . . . . . . . 22
2.7.3 Escape Speed and Orbits . . . . . . . . . . . . . . . . . . . . . . . . 22
2.7.4 Kepler’s Laws . . . . . . . . . . . . . . . . . . . . . . . . . . . . . . 22
2.7.5 Types of Orbits . . . . . . . . . . . . . . . . . . . . . . . . . . . . . 22
2.7.6 Derivation of Vis-viva Equation . . . . . . . . . . . . . . . . . . . 23
2.8 Three Dimensional Particle Dynamics . . . . . . . . . . . . . . . . . . . . 23
2.9 Fluid Dynamics . . . . . . . . . . . . . . . . . . . . . . . . . . . . . . . . . 23
2.9.1 Archimedes’ Principle . . . . . . . . . . . . . . . . . . . . . . . . . 23
2.9.2 Equation of Continuity . . . . . . . . . . . . . . . . . . . . . . . . . 24
2.9.3 Bernoulli’s Equation . . . . . . . . . . . . . . . . . . . . . . . . . . 24
2.10 Non-inertial Reference Frames . . . . . . . . . . . . . . . . . . . . . . . . 24
2.11 Hamiltonian and Lagrangian Formalism . . . . . . . . . . . . . . . . . . . 24
2.11.1 Lagrange’s Function (L) . . . . . . . . . . . . . . . . . . . . . . . . 24
2.11.2 Equations of Motion(Euler-Lagrange Equation) . . . . . . . . . . 24

FT
2.11.3 Hamiltonian . . . . . . . . . . . . . . . . . . . . . . . . . . . . . . . 24

3 Electromagnetism 25
3.1 Electrostatics . . . . . . . . . . . . . . . . . . . . . . . . . . . . . . . . . . . 25
3.1.1 Coulomb’s Law . . . . . . . . . . . . . . . . . . . . . . . . . . . . . 25
3.1.2 Electric Field of a point charge . . . . . . . . . . . . . . . . . . . . 25
3.1.3 Gauss’ Law . . . . . . . . . . . . . . . . . . . . . . . . . . . . . . . 26
3.1.4 Equivalence of Coulomb’s Law and Gauss’ Law . . . . . . . . . . 27
RA
3.1.5 Electric Field due to a line of charge . . . . . . . . . . . . . . . . . 27
3.1.6 Electric Field in a Solid Non-Conducting Sphere . . . . . . . . . . 27
3.1.7 Electric Potential Energy . . . . . . . . . . . . . . . . . . . . . . . . 28
3.1.8 Electric Potential of a Point Charge . . . . . . . . . . . . . . . . . . 28
3.1.9 Electric Potential due to a line charge along axis . . . . . . . . . . 29
3.2 Currents and DC Circuits . . . . . . . . . . . . . . . . . . . . . . . . . . . 30
3.3 Magnetic Fields in Free Space . . . . . . . . . . . . . . . . . . . . . . . . . 30
3.4 Lorentz Force . . . . . . . . . . . . . . . . . . . . . . . . . . . . . . . . . . 30
3.5 Induction . . . . . . . . . . . . . . . . . . . . . . . . . . . . . . . . . . . . . 30
3.6 Maxwell’s Equations and their Applications . . . . . . . . . . . . . . . . . 30
D

3.7 Electromagnetic Waves . . . . . . . . . . . . . . . . . . . . . . . . . . . . . 30


3.8 AC Circuits . . . . . . . . . . . . . . . . . . . . . . . . . . . . . . . . . . . 30
3.9 Magnetic and Electric Fields in Matter . . . . . . . . . . . . . . . . . . . . 30
3.10 Capacitance . . . . . . . . . . . . . . . . . . . . . . . . . . . . . . . . . . . 30
3.11 Energy in a Capacitor . . . . . . . . . . . . . . . . . . . . . . . . . . . . . . 31
3.12 Energy in an Electric Field . . . . . . . . . . . . . . . . . . . . . . . . . . . 31
3.13 Current . . . . . . . . . . . . . . . . . . . . . . . . . . . . . . . . . . . . . . 31
3.14 Current Destiny . . . . . . . . . . . . . . . . . . . . . . . . . . . . . . . . . 31
3.15 Current Density of Moving Charges . . . . . . . . . . . . . . . . . . . . . 31
3.16 Resistance and Ohm’s Law . . . . . . . . . . . . . . . . . . . . . . . . . . 31
3.17 Resistivity and Conductivity . . . . . . . . . . . . . . . . . . . . . . . . . . 31
3.18 Power . . . . . . . . . . . . . . . . . . . . . . . . . . . . . . . . . . . . . . . 32
3.19 Kirchoff’s Loop Rules . . . . . . . . . . . . . . . . . . . . . . . . . . . . . . 32
3.20 Kirchoff’s Junction Rule . . . . . . . . . . . . . . . . . . . . . . . . . . . . 32

David S. Latchman ©2009


Contents 7
3.21 RC Circuits . . . . . . . . . . . . . . . . . . . . . . . . . . . . . . . . . . . . 32
3.22 Maxwell’s Equations . . . . . . . . . . . . . . . . . . . . . . . . . . . . . . 32
3.22.1 Integral Form . . . . . . . . . . . . . . . . . . . . . . . . . . . . . . 32
3.22.2 Differential Form . . . . . . . . . . . . . . . . . . . . . . . . . . . . 33
3.23 Speed of Propagation of a Light Wave . . . . . . . . . . . . . . . . . . . . 33
3.24 Relationship between E and B Fields . . . . . . . . . . . . . . . . . . . . . 33
3.25 Energy Density of an EM wave . . . . . . . . . . . . . . . . . . . . . . . . 33
3.26 Poynting’s Vector . . . . . . . . . . . . . . . . . . . . . . . . . . . . . . . . 33

4 Optics & Wave Phonomena 35


4.1 Wave Properties . . . . . . . . . . . . . . . . . . . . . . . . . . . . . . . . . 35
4.2 Superposition . . . . . . . . . . . . . . . . . . . . . . . . . . . . . . . . . . 35
4.3 Interference . . . . . . . . . . . . . . . . . . . . . . . . . . . . . . . . . . . 35
4.4 Diffraction . . . . . . . . . . . . . . . . . . . . . . . . . . . . . . . . . . . . 35

FT
4.5 Geometrical Optics . . . . . . . . . . . . . . . . . . . . . . . . . . . . . . . 35
4.6 Polarization . . . . . . . . . . . . . . . . . . . . . . . . . . . . . . . . . . . 35
4.7 Doppler Effect . . . . . . . . . . . . . . . . . . . . . . . . . . . . . . . . . . 35
4.8 Snell’s Law . . . . . . . . . . . . . . . . . . . . . . . . . . . . . . . . . . . . 36
4.8.1 Snell’s Law . . . . . . . . . . . . . . . . . . . . . . . . . . . . . . . 36
4.8.2 Critical Angle and Snell’s Law . . . . . . . . . . . . . . . . . . . . 36

5 Thermodynamics & Statistical Mechanics 37


RA
5.1 Laws of Thermodynamics . . . . . . . . . . . . . . . . . . . . . . . . . . . 37
5.2 Thermodynamic Processes . . . . . . . . . . . . . . . . . . . . . . . . . . . 37
5.3 Equations of State . . . . . . . . . . . . . . . . . . . . . . . . . . . . . . . . 37
5.4 Ideal Gases . . . . . . . . . . . . . . . . . . . . . . . . . . . . . . . . . . . . 37
5.5 Kinetic Theory . . . . . . . . . . . . . . . . . . . . . . . . . . . . . . . . . . 37
5.6 Ensembles . . . . . . . . . . . . . . . . . . . . . . . . . . . . . . . . . . . . 37
5.7 Statistical Concepts and Calculation of Thermodynamic Properties . . . 38
5.8 Thermal Expansion & Heat Transfer . . . . . . . . . . . . . . . . . . . . . 38
5.9 Heat Capacity . . . . . . . . . . . . . . . . . . . . . . . . . . . . . . . . . . 38
5.10 Specific Heat Capacity . . . . . . . . . . . . . . . . . . . . . . . . . . . . . 38
D

5.11 Heat and Work . . . . . . . . . . . . . . . . . . . . . . . . . . . . . . . . . 38


5.12 First Law of Thermodynamics . . . . . . . . . . . . . . . . . . . . . . . . . 38
5.12.1 Special Cases to the First Law of Thermodynamics . . . . . . . . 38
5.13 Work done by Ideal Gas at Constant Temperature . . . . . . . . . . . . . 39
5.14 Heat Conduction Equation . . . . . . . . . . . . . . . . . . . . . . . . . . . 39
5.15 Ideal Gas Law . . . . . . . . . . . . . . . . . . . . . . . . . . . . . . . . . . 39
5.16 Stefan-Boltzmann’s Formula . . . . . . . . . . . . . . . . . . . . . . . . . . 40
5.17 RMS Speed of an Ideal Gas . . . . . . . . . . . . . . . . . . . . . . . . . . 40
5.18 Translational Kinetic Energy . . . . . . . . . . . . . . . . . . . . . . . . . . 40
5.19 Internal Energy of a Monatomic gas . . . . . . . . . . . . . . . . . . . . . 40
5.20 Molar Specific Heat at Constant Volume . . . . . . . . . . . . . . . . . . . 40
5.21 Molar Specific Heat at Constant Pressure . . . . . . . . . . . . . . . . . . 41
5.22 Equipartition of Energy . . . . . . . . . . . . . . . . . . . . . . . . . . . . 41
5.23 Adiabatic Expansion of an Ideal Gas . . . . . . . . . . . . . . . . . . . . . 41

©2009 David S. Latchman


8 Contents
5.24 Second Law of Thermodynamics . . . . . . . . . . . . . . . . . . . . . . . 41

6 Quantum Mechanics 43
6.1 Fundamental Concepts . . . . . . . . . . . . . . . . . . . . . . . . . . . . . 43
6.2 Schrödinger Equation . . . . . . . . . . . . . . . . . . . . . . . . . . . . . . 43
6.2.1 Infinite Square Wells . . . . . . . . . . . . . . . . . . . . . . . . . . 44
6.2.2 Harmonic Oscillators . . . . . . . . . . . . . . . . . . . . . . . . . . 45
6.2.3 Finite Square Well . . . . . . . . . . . . . . . . . . . . . . . . . . . 47
6.2.4 Hydrogenic Atoms . . . . . . . . . . . . . . . . . . . . . . . . . . . 48
6.3 Spin . . . . . . . . . . . . . . . . . . . . . . . . . . . . . . . . . . . . . . . . 48
6.4 Angular Momentum . . . . . . . . . . . . . . . . . . . . . . . . . . . . . . 48
6.5 Wave Funtion Symmetry . . . . . . . . . . . . . . . . . . . . . . . . . . . . 48
6.6 Elementary Perturbation Theory . . . . . . . . . . . . . . . . . . . . . . . 48

FT
7 Atomic Physics 49
7.1 Properties of Electrons . . . . . . . . . . . . . . . . . . . . . . . . . . . . . 49
7.2 Bohr Model . . . . . . . . . . . . . . . . . . . . . . . . . . . . . . . . . . . 49
7.3 Energy Quantization . . . . . . . . . . . . . . . . . . . . . . . . . . . . . . 50
7.4 Atomic Structure . . . . . . . . . . . . . . . . . . . . . . . . . . . . . . . . 50
7.5 Atomic Spectra . . . . . . . . . . . . . . . . . . . . . . . . . . . . . . . . . 51
7.5.1 Rydberg’s Equation . . . . . . . . . . . . . . . . . . . . . . . . . . . 51
7.6 Selection Rules . . . . . . . . . . . . . . . . . . . . . . . . . . . . . . . . . . 51
RA
7.7 Black Body Radiation . . . . . . . . . . . . . . . . . . . . . . . . . . . . . . 51
7.7.1 Plank Formula . . . . . . . . . . . . . . . . . . . . . . . . . . . . . 51
7.7.2 Stefan-Boltzmann Formula . . . . . . . . . . . . . . . . . . . . . . 51
7.7.3 Wein’s Displacement Law . . . . . . . . . . . . . . . . . . . . . . . 51
7.7.4 Classical and Quantum Aspects of the Plank Equation . . . . . . 51
7.8 X-Rays . . . . . . . . . . . . . . . . . . . . . . . . . . . . . . . . . . . . . . 52
7.8.1 Bragg Condition . . . . . . . . . . . . . . . . . . . . . . . . . . . . 52
7.8.2 The Compton Effect . . . . . . . . . . . . . . . . . . . . . . . . . . 52
7.9 Atoms in Electric and Magnetic Fields . . . . . . . . . . . . . . . . . . . . 53
7.9.1 The Cyclotron Frequency . . . . . . . . . . . . . . . . . . . . . . . 53
D

7.9.2 Zeeman Effect . . . . . . . . . . . . . . . . . . . . . . . . . . . . . . 54


7.9.3 Franck-Hertz Experiment . . . . . . . . . . . . . . . . . . . . . . . 55

8 Special Relativity 57
8.1 Introductory Concepts . . . . . . . . . . . . . . . . . . . . . . . . . . . . . 57
8.1.1 Postulates of Special Relativity . . . . . . . . . . . . . . . . . . . . 57
8.2 Time Dilation . . . . . . . . . . . . . . . . . . . . . . . . . . . . . . . . . . 57
8.3 Length Contraction . . . . . . . . . . . . . . . . . . . . . . . . . . . . . . . 57
8.4 Simultaneity . . . . . . . . . . . . . . . . . . . . . . . . . . . . . . . . . . . 57
8.5 Energy and Momentum . . . . . . . . . . . . . . . . . . . . . . . . . . . . 58
8.5.1 Relativistic Momentum & Energy . . . . . . . . . . . . . . . . . . 58
8.5.2 Lorentz Transformations (Momentum & Energy) . . . . . . . . . 58
8.5.3 Relativistic Kinetic Energy . . . . . . . . . . . . . . . . . . . . . . . 58
8.5.4 Relativistic Dynamics (Collisions) . . . . . . . . . . . . . . . . . . 58

David S. Latchman ©2009


Contents 9
8.6 Four-Vectors and Lorentz Transformation . . . . . . . . . . . . . . . . . . 59
8.7 Velocity Addition . . . . . . . . . . . . . . . . . . . . . . . . . . . . . . . . 60
8.8 Relativistic Doppler Formula . . . . . . . . . . . . . . . . . . . . . . . . . 60
8.9 Lorentz Transformations . . . . . . . . . . . . . . . . . . . . . . . . . . . . 60
8.10 Space-Time Interval . . . . . . . . . . . . . . . . . . . . . . . . . . . . . . . 61

9 Laboratory Methods 63
9.1 Data and Error Analysis . . . . . . . . . . . . . . . . . . . . . . . . . . . . 63
9.1.1 Addition and Subtraction . . . . . . . . . . . . . . . . . . . . . . . 63
9.1.2 Multiplication and Division . . . . . . . . . . . . . . . . . . . . . . 63
9.1.3 Exponent - (No Error in b) . . . . . . . . . . . . . . . . . . . . . . . 63
9.1.4 Logarithms . . . . . . . . . . . . . . . . . . . . . . . . . . . . . . . 63
9.1.5 Antilogs . . . . . . . . . . . . . . . . . . . . . . . . . . . . . . . . . 64
9.2 Instrumentation . . . . . . . . . . . . . . . . . . . . . . . . . . . . . . . . . 65

FT
9.3 Radiation Detection . . . . . . . . . . . . . . . . . . . . . . . . . . . . . . . 65
9.4 Counting Statistics . . . . . . . . . . . . . . . . . . . . . . . . . . . . . . . 65
9.5 Interaction of Charged Particles with Matter . . . . . . . . . . . . . . . . 65
9.6 Lasers and Optical Interferometers . . . . . . . . . . . . . . . . . . . . . . 65
9.7 Dimensional Analysis . . . . . . . . . . . . . . . . . . . . . . . . . . . . . 66
9.8 Fundamental Applications of Probability and Statistics . . . . . . . . . . 66

10 GR8677 Exam Solutions 67


RA
10.1 Motion of Rock under Drag Force . . . . . . . . . . . . . . . . . . . . . . . 67
10.2 Satellite Orbits . . . . . . . . . . . . . . . . . . . . . . . . . . . . . . . . . . 68
10.3 Speed of Light in a Dielectric Medium . . . . . . . . . . . . . . . . . . . . 68
10.4 Wave Equation . . . . . . . . . . . . . . . . . . . . . . . . . . . . . . . . . 68
10.5 Inelastic Collision and Putty Spheres . . . . . . . . . . . . . . . . . . . . . 69
10.6 Motion of a Particle along a Track . . . . . . . . . . . . . . . . . . . . . . . 70
10.7 Resolving Force Components . . . . . . . . . . . . . . . . . . . . . . . . . 70
10.8 Nail being driven into a block of wood . . . . . . . . . . . . . . . . . . . . 71
10.9 Current Density . . . . . . . . . . . . . . . . . . . . . . . . . . . . . . . . . 71
10.10Charge inside an Isolated Sphere . . . . . . . . . . . . . . . . . . . . . . . 72
D

10.11Vector Identities and Maxwell’s Laws . . . . . . . . . . . . . . . . . . . . 73


10.12Doppler Equation (Non-Relativistic) . . . . . . . . . . . . . . . . . . . . . 73
10.13Vibrating Interference Pattern . . . . . . . . . . . . . . . . . . . . . . . . . 73
10.14Specific Heat at Constant Pressure and Volume . . . . . . . . . . . . . . . 73
10.15Helium atoms in a box . . . . . . . . . . . . . . . . . . . . . . . . . . . . . 74
10.16The Muon . . . . . . . . . . . . . . . . . . . . . . . . . . . . . . . . . . . . 75
10.17Radioactive Decay . . . . . . . . . . . . . . . . . . . . . . . . . . . . . . . 75
10.18Schrödinger’s Equation . . . . . . . . . . . . . . . . . . . . . . . . . . . . 75
10.19Energy Levels of Bohr’s Hydrogen Atom . . . . . . . . . . . . . . . . . . 76
10.20Relativistic Energy . . . . . . . . . . . . . . . . . . . . . . . . . . . . . . . 76
10.21Space-Time Interval . . . . . . . . . . . . . . . . . . . . . . . . . . . . . . . 77
10.22Lorentz Transformation of the EM field . . . . . . . . . . . . . . . . . . . 77
10.23Conductivity of a Metal and Semi-Conductor . . . . . . . . . . . . . . . . 78
10.24Charging a Battery . . . . . . . . . . . . . . . . . . . . . . . . . . . . . . . 78

©2009 David S. Latchman


10 Contents
10.25Lorentz Force on a Charged Particle . . . . . . . . . . . . . . . . . . . . . 78
10.26K-Series X-Rays . . . . . . . . . . . . . . . . . . . . . . . . . . . . . . . . . 79
10.27Electrons and Spin . . . . . . . . . . . . . . . . . . . . . . . . . . . . . . . 79
10.28Normalizing a wavefunction . . . . . . . . . . . . . . . . . . . . . . . . . 81
10.29Right Hand Rule . . . . . . . . . . . . . . . . . . . . . . . . . . . . . . . . 81
10.30Electron Configuration of a Potassium atom . . . . . . . . . . . . . . . . . 81
10.31Photoelectric Effect I . . . . . . . . . . . . . . . . . . . . . . . . . . . . . . 82
10.32Photoelectric Effect II . . . . . . . . . . . . . . . . . . . . . . . . . . . . . . 82
10.33Photoelectric Effect III . . . . . . . . . . . . . . . . . . . . . . . . . . . . . 82
10.34Potential Energy of a Body . . . . . . . . . . . . . . . . . . . . . . . . . . . 82
10.35Hamiltonian of a Body . . . . . . . . . . . . . . . . . . . . . . . . . . . . . 83
10.36Principle of Least Action . . . . . . . . . . . . . . . . . . . . . . . . . . . . 83
10.37Tension in a Conical Pendulum . . . . . . . . . . . . . . . . . . . . . . . . 83
10.38Diode OR-gate . . . . . . . . . . . . . . . . . . . . . . . . . . . . . . . . . . 84

FT
10.39Gain of an Amplifier vs. Angular Frequency . . . . . . . . . . . . . . . . 84
10.40Counting Statistics . . . . . . . . . . . . . . . . . . . . . . . . . . . . . . . 84
10.41Binding Energy per Nucleon . . . . . . . . . . . . . . . . . . . . . . . . . . 85
10.42Scattering Cross Section . . . . . . . . . . . . . . . . . . . . . . . . . . . . 85
10.43Coupled Oscillators . . . . . . . . . . . . . . . . . . . . . . . . . . . . . . . 85
10.43.1 Calculating the modes of oscillation . . . . . . . . . . . . . . . . . 85
10.44Collision with a Rod . . . . . . . . . . . . . . . . . . . . . . . . . . . . . . 86
10.45Compton Wavelength . . . . . . . . . . . . . . . . . . . . . . . . . . . . . 87
RA
10.46Stefan-Boltzmann’s Equation . . . . . . . . . . . . . . . . . . . . . . . . . 87
10.47Franck-Hertz Experiment . . . . . . . . . . . . . . . . . . . . . . . . . . . 87
10.48Selection Rules for Electronic Transitions . . . . . . . . . . . . . . . . . . 87
10.49The Hamilton Operator . . . . . . . . . . . . . . . . . . . . . . . . . . . . 88
10.50Hall Effect . . . . . . . . . . . . . . . . . . . . . . . . . . . . . . . . . . . . 88
10.51Debye and Einstein Theories to Specific Heat . . . . . . . . . . . . . . . . 89
10.52Potential inside a Hollow Cube . . . . . . . . . . . . . . . . . . . . . . . . 90
10.53EM Radiation from Oscillating Charges . . . . . . . . . . . . . . . . . . . 90
10.54Polarization Charge Density . . . . . . . . . . . . . . . . . . . . . . . . . . 90
10.55Kinetic Energy of Electrons in Metals . . . . . . . . . . . . . . . . . . . . . 91
D

10.56Expectation or Mean Value . . . . . . . . . . . . . . . . . . . . . . . . . . . 91


10.57Eigenfuction of Wavefunction . . . . . . . . . . . . . . . . . . . . . . . . . 91
10.58Holograms . . . . . . . . . . . . . . . . . . . . . . . . . . . . . . . . . . . . 92
10.59Group Velocity of a Wave . . . . . . . . . . . . . . . . . . . . . . . . . . . 93
10.60Potential Energy and Simple Harmonic Motion . . . . . . . . . . . . . . . 93
10.61Rocket Equation I . . . . . . . . . . . . . . . . . . . . . . . . . . . . . . . . 94
10.62Rocket Equation II . . . . . . . . . . . . . . . . . . . . . . . . . . . . . . . 94
10.63Surface Charge Density . . . . . . . . . . . . . . . . . . . . . . . . . . . . . 94
10.64Maximum Power Theorem . . . . . . . . . . . . . . . . . . . . . . . . . . . 95
10.65Magnetic Field far away from a Current carrying Loop . . . . . . . . . . 95
10.66Maxwell’s Relations . . . . . . . . . . . . . . . . . . . . . . . . . . . . . . . 96
10.67Partition Functions . . . . . . . . . . . . . . . . . . . . . . . . . . . . . . . 97
10.68Particle moving at Light Speed . . . . . . . . . . . . . . . . . . . . . . . . 97
10.69Car and Garage I . . . . . . . . . . . . . . . . . . . . . . . . . . . . . . . . 97

David S. Latchman ©2009


Contents 11
10.70Car and Garage II . . . . . . . . . . . . . . . . . . . . . . . . . . . . . . . . 97
10.71Car and Garage III . . . . . . . . . . . . . . . . . . . . . . . . . . . . . . . 98
10.72Refrective Index of Rock Salt and X-rays . . . . . . . . . . . . . . . . . . . 98
10.73Thin Flim Non-Reflective Coatings . . . . . . . . . . . . . . . . . . . . . . 99
10.74Law of Malus . . . . . . . . . . . . . . . . . . . . . . . . . . . . . . . . . . 99
10.75Geosynchronous Satellite Orbit . . . . . . . . . . . . . . . . . . . . . . . . 100
10.76Hoop Rolling down and Inclined Plane . . . . . . . . . . . . . . . . . . . 100
10.77Simple Harmonic Motion . . . . . . . . . . . . . . . . . . . . . . . . . . . 101
10.78Total Energy between Two Charges . . . . . . . . . . . . . . . . . . . . . . 101
10.79Maxwell’s Equations and Magnetic Monopoles . . . . . . . . . . . . . . . 102
10.80Gauss’ Law . . . . . . . . . . . . . . . . . . . . . . . . . . . . . . . . . . . . 102
10.81Biot-Savart Law . . . . . . . . . . . . . . . . . . . . . . . . . . . . . . . . . 103
10.82Zeeman Effect and the emission spectrum of atomic gases . . . . . . . . 104
10.83Spectral Lines in High Density and Low Density Gases . . . . . . . . . . 105

FT
10.84Term Symbols & Spectroscopic Notation . . . . . . . . . . . . . . . . . . . 105
10.85Photon Interaction Cross Sections for Pb . . . . . . . . . . . . . . . . . . . 106
10.86The Ice Pail Experiment . . . . . . . . . . . . . . . . . . . . . . . . . . . . 106
10.87Equipartition of Energy and Diatomic Molecules . . . . . . . . . . . . . . 106
10.88Fermion and Boson Pressure . . . . . . . . . . . . . . . . . . . . . . . . . . 107
10.89Wavefunction of Two Identical Particles . . . . . . . . . . . . . . . . . . . 107
10.90Energy Eigenstates . . . . . . . . . . . . . . . . . . . . . . . . . . . . . . . 108
10.91Bragg’s Law . . . . . . . . . . . . . . . . . . . . . . . . . . . . . . . . . . . 108
RA
10.92Selection Rules for Electronic Transitions . . . . . . . . . . . . . . . . . . 109
10.93Moving Belt Sander on a Rough Plane . . . . . . . . . . . . . . . . . . . . 109
10.94RL Circuits . . . . . . . . . . . . . . . . . . . . . . . . . . . . . . . . . . . . 110
10.95Carnot Cycles . . . . . . . . . . . . . . . . . . . . . . . . . . . . . . . . . . 111
10.96First Order Perturbation Theory . . . . . . . . . . . . . . . . . . . . . . . . 113
10.97Colliding Discs and the Conservation of Angular Momentum . . . . . . 114
10.98Electrical Potential of a Long Thin Rod . . . . . . . . . . . . . . . . . . . . 114
10.99Ground State of a Positronium Atom . . . . . . . . . . . . . . . . . . . . . 115
10.100The Pinhole Camera . . . . . . . . . . . . . . . . . . . . . . . . . . . . . . 115
D

A Constants & Important Equations 117


A.1 Constants . . . . . . . . . . . . . . . . . . . . . . . . . . . . . . . . . . . . . 117
A.2 Vector Identities . . . . . . . . . . . . . . . . . . . . . . . . . . . . . . . . . 117
A.2.1 Triple Products . . . . . . . . . . . . . . . . . . . . . . . . . . . . . 117
A.2.2 Product Rules . . . . . . . . . . . . . . . . . . . . . . . . . . . . . . 118
A.2.3 Second Derivatives . . . . . . . . . . . . . . . . . . . . . . . . . . . 118
A.3 Commutators . . . . . . . . . . . . . . . . . . . . . . . . . . . . . . . . . . 118
A.3.1 Lie-algebra Relations . . . . . . . . . . . . . . . . . . . . . . . . . . 118
A.3.2 Canonical Commutator . . . . . . . . . . . . . . . . . . . . . . . . 118
A.3.3 Kronecker Delta Function . . . . . . . . . . . . . . . . . . . . . . . 118
A.4 Linear Algebra . . . . . . . . . . . . . . . . . . . . . . . . . . . . . . . . . . 118
A.4.1 Vectors . . . . . . . . . . . . . . . . . . . . . . . . . . . . . . . . . . 118

©2009 David S. Latchman


12 Contents

FT
RA
D

David S. Latchman ©2009


Chapter 2

Classical Mechanics

2.1 Kinematics
2.1.1 Linear Motion
Average Velocity

Instantaneous Velocity
v=
∆tFT
∆x x2 − x1
=
t2 − t1
(2.1)
RA
∆x dx
v = lim = = v(t) (2.2)
∆t→0 ∆t dt

Kinematic Equations of Motion


The basic kinematic equations of motion under constant acceleration, a, are
v = v0 + at (2.3)
v2 = v20 + 2a (x − x0 ) (2.4)
D

1
x − x0 = v0 t + at2 (2.5)
2
1
x − x0 = (v + v0 ) t (2.6)
2

2.1.2 Circular Motion


In the case of Uniform Circular Motion, for a particle to move in a circular path, a
radial acceleration must be applied. This acceleration is known as the Centripetal
Acceleration

Centripetal Acceleration
v2
a= (2.7)
r
14 Classical Mechanics
Angular Velocity
v
ω= (2.8)
r
We can write (Equation 2.7) in terms of ω

a = ω2 r (2.9)

Rotational Equations of Motion

The equations of motion under a constant angular acceleration, α, are

ω = ω0 + αt (2.10)
ω + ω0

FT
θ= t (2.11)
2
1
θ = ω0 t + αt2 (2.12)
2
ω = ω0 + 2αθ
2 2
(2.13)

2.2 Newton’s Laws


RA
2.2.1 Newton’s Laws of Motion
First Law A body continues in its state of rest or of uniform motion unless acted upon
by an external unbalanced force.

Second Law The net force on a body is proportional to its rate of change of momentum.

dp
F= = ma (2.14)
dt
D

Third Law When a particle A exerts a force on another particle B, B simultaneously


exerts a force on A with the same magnitude in the opposite direction.

FAB = −FBA (2.15)

2.2.2 Momentum
p = mv (2.16)

2.2.3 Impulse
w
∆p = J = Fdt = Favg dt (2.17)

David S. Latchman ©2009


Work & Energy 15
2.3 Work & Energy
2.3.1 Kinetic Energy
1
K ≡ mv2 (2.18)
2

2.3.2 The Work-Energy Theorem


The net Work done is given by
Wnet = K f − Ki (2.19)

2.3.3 Work done under a constant Force

FT
The work done by a force can be expressed as

W = F∆x (2.20)

In three dimensions, this becomes

W = F · ∆r = F∆r cos θ (2.21)

For a non-constant force, we have


RA
wx f
W= F(x)dx (2.22)
xi

2.3.4 Potential Energy


The Potential Energy is
dU(x)
F(x) = − (2.23)
dx
for conservative forces, the potential energy is
D

wx
U(x) = U0 − F(x0 )dx0 (2.24)
x0

2.3.5 Hooke’s Law


F = −kx (2.25)
where k is the spring constant.

2.3.6 Potential Energy of a Spring


1
U(x) = kx2 (2.26)
2

©2009 David S. Latchman


16 Classical Mechanics
2.4 Oscillatory Motion
2.4.1 Equation for Simple Harmonic Motion
x(t) = A sin (ωt + δ) (2.27)
where the Amplitude, A, measures the displacement from equilibrium, the phase, δ, is
the angle by which the motion is shifted from equilibrium at t = 0.

2.4.2 Period of Simple Harmonic Motion



T= (2.28)
ω

2.4.3 Total Energy of an Oscillating System

FT
Given that
x = A sin (ωt + δ) (2.29)
and that the Total Energy of a System is
E = KE + PE (2.30)
The Kinetic Energy is
RA
1
KE = mv2
2
1 dx
= m
2 dt
1
= mA2 ω2 cos2 (ωt + δ) (2.31)
2
The Potential Energy is
1
U = kx2
2
D

1
= kA2 sin2 (ωt + δ) (2.32)
2
Adding (Equation 2.31) and (Equation 2.32) gives
1
E = kA2 (2.33)
2

2.4.4 Damped Harmonic Motion


dx
Fd = −bv = −b (2.34)
dt
where b is the damping coefficient. The equation of motion for a damped oscillating
system becomes
dx d2 x
− kx − b = m 2 (2.35)
dt dt

David S. Latchman ©2009


Oscillatory Motion 17
Solving(Equation 2.35) goves
x = Ae−αt sin (ω0 t + δ) (2.36)
We find that
b
α= (2.37)
2m
r
k b2
ω0 = −
m 4m2
r
b2
= ω20 −
4m2
q
= ω20 − α2 (2.38)

FT
2.4.5 Small Oscillations
The Energy of a system is
1
E = K + V(x) = mv(x)2 + V(x) (2.39)
2
We can solve for v(x), r
2
v(x) = (E − V(x)) (2.40)
RA
m
where E ≥ V(x) Let the particle move in the potential valley, x1 ≤ x ≤ x2 , the potential
can be approximated by the Taylor Expansion
" # " 2 #
dV(x) 1 2 d V(x)
V(x) = V(xe ) + (x − xe ) + (x − xe ) + ··· (2.41)
dx x=xe 2 dx2 x=xe

At the points of inflection, the derivative dV/dx is zero and d2 V/dx2 is positive. This
means that the potential energy for small oscillations becomes
1
V(x) u V(xe ) + k(x − xe )2 (2.42)
D

2
where " #
d2 V(x)
k≡ ≥0 (2.43)
dx2 x=xe
As V(xe ) is constant, it has no consequences to physical motion and can be dropped.
We see that Equation 2.42 is that of simple harmonic motion.

2.4.6 Coupled Harmonic Oscillators


Consider the case of a simple pendulum of length, `, and the mass of the bob is m1 .
For small displacements, the equation of motion is
θ̈ + ω0 θ = 0 (2.44)
1
Add figure with coupled pendulum-spring system

©2009 David S. Latchman


18 Classical Mechanics
We can express this in cartesian coordinates, x and y, where

x = ` cos θ ≈ ` (2.45)
y = ` sin θ ≈ `θ (2.46)

Equation 2.44 becomes


ÿ + ω0 y = 0 (2.47)
This is the equivalent to the mass-spring system where the spring constant is
mg
k = mω20 = (2.48)
`
This allows us to to create an equivalent three spring system to our coupled pendulum
system. The equations of motion can be derived from the Lagrangian, where

FT
L=T−V
1 2 1 2 1 2 1 2 1 2
 
= m ẏ1 + m ẏ2 − ky1 + κ y2 − y1 + ky2
2 2 2 2 2
1  2  1   2 
= m y˙1 + y˙2 2 − k y21 + y22 + κ y2 − y1 (2.49)
2 2
We can find the equations of motion of our system

d ∂L ∂L
!
RA
= (2.50)
dt ∂ ẏn ∂yn

The equations of motion are

m ÿ1 = −ky1 + κ y2 − y1

(2.51)
m ÿ2 = −ky2 + κ y2 − y1

(2.52)

We assume solutions for the equations of motion to be of the form

y1 = cos(ωt + δ1 ) y2 = B cos(ωt + δ2 )
(2.53)
D

ÿ1 = −ωy1 ÿ2 = −ωy2

Substituting the values for ÿ1 and ÿ2 into the equations of motion yields
 
k + κ − mω2 y1 − κy2 = 0 (2.54)
 
−κy1 + k + κ − mω2 y2 = 0 (2.55)

We can get solutions from solving the determinant of the matrix

k + κ − mω2

−κ
=0 (2.56)

k + κ − mω2

−κ

Solving the determinant gives


 2  
mω2 − 2mω2 (k + κ) + k2 + 2kκ = 0 (2.57)

David S. Latchman ©2009


Oscillatory Motion 19
This yields
g

k
=


`

ω2 =  m

(2.58)

k + 2κ g 2κ
= +



m ` m

We can now determine exactly how the masses move with each mode by substituting
ω2 into the equations of motion. Where

k
ω2 = We see that
m
k + κ − mω2 = κ (2.59)
Substituting this into the equation of motion yields

y1 = y2 (2.60)

ω2 =
its absence in our result.
k+κ
m
We see that
FT
We see that the masses move in phase with each other. You will also notice
the absense of the spring constant term, κ, for the connecting spring. As the
masses are moving in step, the spring isn’t stretching or compressing and hence
RA
k + κ − mω2 = −κ (2.61)
Substituting this into the equation of motion yields

y1 = −y2 (2.62)

Here the masses move out of phase with each other. In this case we see the
presence of the spring constant, κ, which is expected as the spring playes a role.
It is being stretched and compressed as our masses oscillate.
D

2.4.7 Doppler Effect


The Doppler Effect is the shift in frequency and wavelength of waves that results from
a source moving with respect to the medium, a receiver moving with respect to the
medium or a moving medium.

Moving Source If a source is moving towards an observer, then in one period, τ0 , it


moves a distance of vs τ0 = vs / f0 . The wavelength is decreased by

vs v − vs
λ0 = λ − − (2.63)
f0 f0

The frequency change is


v v
 
f = 0 = f0
0
(2.64)
λ v − vs

©2009 David S. Latchman


20 Classical Mechanics
Moving Observer As the observer moves, he will measure the same wavelength, λ, as
if at rest but will see the wave crests pass by more quickly. The observer measures
a modified wave speed.
v0 = v + |vr | (2.65)
The modified frequency becomes
v0 vr
 
f =
0
= f0 1 + (2.66)
λ v
Moving Source and Moving Observer We can combine the above two equations
v − vs
λ0 = (2.67)
f0
v0 = v − vr (2.68)
To give a modified frequency of

FT
v0 v − vr
 
f = 0 =
0
f0 (2.69)
λ v − vs

2.5 Rotational Motion about a Fixed Axis


2.5.1 Moment of Inertia
Z
RA
I= R2 dm (2.70)

2.5.2 Rotational Kinetic Energy


1
K = Iω2 (2.71)
2

2.5.3 Parallel Axis Theorem


I = Icm + Md2 (2.72)
D

2.5.4 Torque

τ=r×F (2.73)
τ = Iα (2.74)
where α is the angular acceleration.

2.5.5 Angular Momentum


L = Iω (2.75)
we can find the Torque
dL
τ= (2.76)
dt

David S. Latchman ©2009


Dynamics of Systems of Particles 21
2.5.6 Kinetic Energy in Rolling
With respect to the point of contact, the motion of the wheel is a rotation about the
point of contact. Thus
1
K = Krot = Icontact ω2 (2.77)
2
Icontact can be found from the Parallel Axis Theorem.
Icontact = Icm + MR2 (2.78)
Substitute (Equation 2.77) and we have
1 
K= Icm + MR2 ω2
2
1 1
= Icm ω2 + mv2 (2.79)
2 2

FT
The kinetic energy of an object rolling without slipping is the sum of hte kinetic energy
of rotation about its center of mass and the kinetic energy of the linear motion of the
object.

2.6 Dynamics of Systems of Particles


2.6.1 Center of Mass of a System of Particles
RA
Position Vector of a System of Particles
m1 r1 + m2 r2 + m3 r3 + · · · + mN rN
R= (2.80)
M

Velocity Vector of a System of Particles

dR
V=
dt
m1 v1 + m2 v2 + m3 v3 + · · · + mN vN
= (2.81)
D

Acceleration Vector of a System of Particles

dV
A=
dt
m1 a1 + m2 a2 + m3 a3 + · · · + mN aN
= (2.82)
M

2.7 Central Forces and Celestial Mechanics


2.7.1 Newton’s Law of Universal Gravitation
GMm


F=− r̂ (2.83)
r2

©2009 David S. Latchman


22 Classical Mechanics
2.7.2 Potential Energy of a Gravitational Force
GMm
U(r) = − (2.84)
r

2.7.3 Escape Speed and Orbits


The energy of an orbiting body is

E=T+U
1 GMm
= mv2 − (2.85)
2 r
The escape speed becomes
1 GMm

FT
E = mv2esc − =0 (2.86)
2 RE
Solving for vesc we find r
2GM
vesc = (2.87)
Re

2.7.4 Kepler’s Laws


RA
First Law The orbit of every planet is an ellipse with the sun at a focus.
Second Law A line joining a planet and the sun sweeps out equal areas during equal
intervals of time.
Third Law The square of the orbital period of a planet is directly proportional to the
cube of the semi-major axis of its orbit.

T2
=C (2.88)
R3
where C is a constant whose value is the same for all planets.
D

2.7.5 Types of Orbits


The Energy of an Orbiting Body is defined in (Equation 2.85), we can classify orbits by
their eccentricities.
Circular Orbit A circular orbit occurs when there is an eccentricity of 0 and the orbital
energy is less than 0. Thus
1 2 GM
v − =E<0 (2.89)
2 r
The Orbital Velocity is r
GM
v= (2.90)
r

David S. Latchman ©2009


Three Dimensional Particle Dynamics 23
Elliptic Orbit An elliptic orbit occurs when the eccentricity is between 0 and 1 but the
specific energy is negative, so the object remains bound.
r
2 1
 
v= GM − (2.91)
r a
where a is the semi-major axis

Parabolic Orbit A Parabolic Orbit occurs when the eccentricity is equal to 1 and the
orbital velocity is the escape velocity. This orbit is not bounded. Thus

1 2 GM
v − =E=0 (2.92)
2 r
The Orbital Velocity is

FT
r
2GM
v = vesc = (2.93)
r
Hyperbolic Orbit In the Hyperbolic Orbit, the eccentricity is greater than 1 with an
orbital velocity in excess of the escape velocity. This orbit is also not bounded.
r
GM
v∞ = (2.94)
a
RA
2.7.6 Derivation of Vis-viva Equation
The total energy of a satellite is

1 GMm
E = mv2 − (2.95)
2 r
For an elliptical or circular orbit, the specific energy is

GMm
E=−
D

(2.96)
2a
Equating we get
2 1
 
v = GM −
2
(2.97)
r a

2.8 Three Dimensional Particle Dynamics

2.9 Fluid Dynamics


2.9.1 Archimedes’ Principle
When an object is fully or partially immersed, the buoyant force is equal to the weight
of fluid displaced.

©2009 David S. Latchman


24 Classical Mechanics
2.9.2 Equation of Continuity
ρ1 v1 A1 = ρ2 v2 A2 (2.98)

2.9.3 Bernoulli’s Equation


1
P + ρv2 + ρgh = a constant (2.99)
2

2.10 Non-inertial Reference Frames

2.11 Hamiltonian and Lagrangian Formalism


2.11.1 Lagrange’s Function (L)

Coordinates.

2.11.2 FT
L=T−V

Equations of Motion(Euler-Lagrange Equation)


∂L d ∂L
(2.100)
where T is the Kinetic Energy and V is the Potential Energy in terms of Generalized
RA
!
= (2.101)
∂q dt ∂q̇

2.11.3 Hamiltonian

H =T+V
= pq̇ − L(q, q̇) (2.102)

where
D

∂H
= q̇ (2.103)
∂p
∂H ∂L
=−
∂q ∂x
= −ṗ (2.104)

David S. Latchman ©2009


Chapter 3

Electromagnetism

3.1 Electrostatics
3.1.1 Coulomb’s Law

F12 =
1 FT
The force between two charged particles, q1 and q2 is defined by Coulomb’s Law.

q1 q2
4π0 r212
!
r̂12 (3.1)
RA
where 0 is the permitivitty of free space, where

0 = 8.85 × 10−12 C2 N.m2 (3.2)

3.1.2 Electric Field of a point charge


The electric field is defined by mesuring the magnitide and direction of an electric
force, F, acting on a test charge, q0 .
D

F
E≡ (3.3)
q0

The Electric Field of a point charge, q is

1 q
E= r̂ (3.4)
4π0 r2

In the case of multiple point charges, qi , the electric field becomes

n
1 X qi
E(r) = r̂i (3.5)
4π0 i=1 r2i
26 Electromagnetism
Electric Fields and Continuous Charge Distributions
If a source is distributed continuously along a region of space, Equation 3.5 becomes
Z
1 1
E(r) = r̂dq (3.6)
4π0 r2

If the charge was distributed along a line with linear charge density, λ,
dq
λ= (3.7)
dx
The Electric Field of a line charge becomes

λ
Z
1
E(r) = r̂dx (3.8)
4π0 r2

FT
line

In the case where the charge is distributed along a surface, the surface charge density
is, σ
Q dq
σ= = (3.9)
A dA
The electric field along the surface becomes

σ
Z
1
RA
E(r) = r̂dA (3.10)
4π0 r2
Surface

In the case where the charge is distributed throughout a volume, V, the volume charge
density is
Q dq
ρ= = (3.11)
V dV
The Electric Field is
ρ
Z
1
E(r) = r̂dV (3.12)
4π0 r2
D

Volume

3.1.3 Gauss’ Law


The electric field through a surface is
I I
Φ= dΦ = E · dA (3.13)
surface S surface S

The electric flux through a closed surface encloses a net charge.


I
Q
E · dA = (3.14)
0

where Q is the charge enclosed by our surface.

David S. Latchman ©2009


Electrostatics 27
3.1.4 Equivalence of Coulomb’s Law and Gauss’ Law
The total flux through a sphere is
I
q
E · dA = E(4πr2 ) = (3.15)
0

From the above, we see that the electric field is


q
E= (3.16)
4π0 r2

3.1.5 Electric Field due to a line of charge


Consider an infinite rod of constant charge density, λ. The flux through a Gaussian

FT
cylinder enclosing the line of charge is
Z Z Z
Φ= E · dA + E · dA + E · dA (3.17)
top surface bottom surface side surface

At the top and bottom surfaces, the electric field is perpendicular to the area vector, so
for the top and bottom surfaces,
E · dA = 0 (3.18)
RA
At the side, the electric field is parallel to the area vector, thus

E · dA = EdA (3.19)

Thus the flux becomes, Z Z


Φ= E · dA = E dA (3.20)
side sirface

The area in this case is the surface area of the side of the cylinder, 2πrh.
D

Φ = 2πrhE (3.21)

Applying Gauss’ Law, we see that Φ = q/0 . The electric field becomes

λ
E= (3.22)
2π0 r

3.1.6 Electric Field in a Solid Non-Conducting Sphere


Within our non-conducting sphere or radius, R, we will assume that the total charge,
Q is evenly distributed throughout the sphere’s volume. So the charge density of our
sphere is
Q Q
ρ= = 4 (3.23)
V 3
πR 3

©2009 David S. Latchman


28 Electromagnetism
The Electric Field due to a charge Q is

Q
E= (3.24)
4π0 r2
As the charge is evenly distributed throughout the sphere’s volume we can say that
the charge density is
dq = ρdV (3.25)
where dV = 4πr2 dr. We can use this to determine the field inside the sphere by
summing the effect of infinitesimally thin spherical shells
Z E Z r
dq
E= dE = 2
0 0 4πr
ρ
Z r
=

FT
dr
0 0
Qr
= 4 (3.26)
3
π0 R3

3.1.7 Electric Potential Energy


1
U(r) = qq0 r (3.27)
4π0
RA
3.1.8 Electric Potential of a Point Charge
The electrical potential is the potential energy per unit charge that is associated with a
static electrical field. It can be expressed thus

U(r) = qV(r) (3.28)

And we can see that


1 q
V(r) = (3.29)
D

4π0 r
A more proper definition that includes the electric field, E would be
Z
V(r) = − E · d` (3.30)
C

where C is any path, starting at a chosen point of zero potential to our desired point.
The difference between two potentials can be expressed such
Z b Z a
V(b) − V(a) = − E · d` + E · d`
Z b
=− E · d` (3.31)
a

David S. Latchman ©2009


Electrostatics 29
This can be further expressed
Z b
V(b) − V(a) = (∇V) · d` (3.32)
a

And we can show that


E = −∇V (3.33)

3.1.9 Electric Potential due to a line charge along axis


Let us consider a rod of length, `, with linear charge density, λ. The Electrical Potential
due to a continuous distribution is
Z Z
1 dq

FT
V= dV = (3.34)
4π0 r

The charge density is


dq = λdx (3.35)
Substituting this into the above equation, we get the electrical potential at some distance
x along the rod’s axis, with the origin at the start of the rod.
RA
1 dq
dV =
4π0 x
1 λdx
= (3.36)
4π0 x

This becomes
λ x2
 
V= ln (3.37)
4π0 x1
where x1 and x2 are the distances from O, the end of the rod.
Now consider that we are some distance, y, from the axis of the rod of length, `. We
D

again look at Equation 3.34, where r is the distance of the point P from the rod’s axis.
Z
1 dq
V=
4π0 r
`
λdx
Z
1
= 1
4π0 0 x2 + y2 2
λ
   12 `
= ln x + x2 + y2
4π0 0
λ  12 
 
= ln ` + `2 + y2 − ln y
4π0
 1 
λ  ` + `2 + y2 2 
= ln   (3.38)
4π0 d


©2009 David S. Latchman


30 Electromagnetism
3.2 Currents and DC Circuits
2

3.3 Magnetic Fields in Free Space


3

3.4 Lorentz Force


4

3.5
5

3.6
Induction

FT
Maxwell’s Equations and their Applications
RA
6

3.7 Electromagnetic Waves


7
D

3.8 AC Circuits
8

3.9 Magnetic and Electric Fields in Matter


9

3.10 Capacitance
Q = CV (3.39)

David S. Latchman ©2009


Energy in a Capacitor 31
3.11 Energy in a Capacitor

Q2
U=
2C
CV 2
=
2
QV
= (3.40)
2

3.12 Energy in an Electric Field


U 0 E2
u≡ = (3.41)

FT
volume 2

3.13 Current
dQ
I≡ (3.42)
dt

3.14 Current Destiny


RA
Z
I= J · dA (3.43)
A

3.15 Current Density of Moving Charges


I
J= = ne qvd (3.44)
A
D

3.16 Resistance and Ohm’s Law


V
R≡ (3.45)
I

3.17 Resistivity and Conductivity


L
R=ρ (3.46)
A

E = ρJ (3.47)

J = σE (3.48)

©2009 David S. Latchman


32 Electromagnetism
3.18 Power
P = VI (3.49)

3.19 Kirchoff’s Loop Rules


Write Here

3.20 Kirchoff’s Junction Rule


Write Here

3.21 RC Circuits

FT
E − IR −
Q
C
=0 (3.50)
RA
3.22 Maxwell’s Equations
3.22.1 Integral Form
Gauss’ Law for Electric Fields
w Q
E · dA = (3.51)
0
closed surface
D

Gauss’ Law for Magnetic Fields


w
B · dA = 0 (3.52)
closed surface

Ampère’s Law
z d w
B · ds = µ0 I + µ0 0 E · dA (3.53)
dt
surface

Faraday’s Law
z d w
E · ds = − B · dA (3.54)
dt
surface

David S. Latchman ©2009


Speed of Propagation of a Light Wave 33
3.22.2 Differential Form
Gauss’ Law for Electric Fields
ρ
∇·E= (3.55)
0
Gauss’ Law for Magnetism
∇·B=0 (3.56)

Ampère’s Law
∂E
∇ × B = µ0 J + µ0 0 (3.57)
∂t
Faraday’s Law
∂B
∇·E=− (3.58)
∂t

3.23 Speed of Propagation of a Light Wave

In a material with dielectric constant, κ,


c= √


FT
c
1
µ0 0
(3.59)
RA
c κ= (3.60)
n
where n is the refractive index.

3.24 Relationship between E and B Fields

E = cB (3.61)
E·B=0 (3.62)
D

3.25 Energy Density of an EM wave


!
1 B2
u= + 0 E2 (3.63)
2 µ0

3.26 Poynting’s Vector


1
S= E×B (3.64)
µ0

©2009 David S. Latchman


34 Electromagnetism

FT
RA
D

David S. Latchman ©2009


Chapter 4

Optics & Wave Phonomena

4.1 Wave Properties


1

4.2
2
Superposition
FT
RA
4.3 Interference
3

4.4 Diffraction
4
D

4.5 Geometrical Optics


5

4.6 Polarization
6

4.7 Doppler Effect


7
36 Optics & Wave Phonomena
4.8 Snell’s Law
4.8.1 Snell’s Law
n1 sin θ1 = n2 sin θ2 (4.1)

4.8.2 Critical Angle and Snell’s Law


The critical angle, θc , for the boundary seperating two optical media is the smallest
angle of incidence, in the medium of greater index, for which light is totally refelected.
From Equation 4.1, θ1 = 90 and θ2 = θc and n2 > n1 .

n1 sin 90 = n2 sinθc
n1
sin θc = (4.2)

FT
n2
RA
D

David S. Latchman ©2009


Chapter 5

Thermodynamics & Statistical


Mechanics

5.1
1

5.2
FT
Laws of Thermodynamics

Thermodynamic Processes
RA
2

5.3 Equations of State


3
D

5.4 Ideal Gases


4

5.5 Kinetic Theory


5

5.6 Ensembles
6
38 Thermodynamics & Statistical Mechanics
5.7 Statistical Concepts and Calculation of Thermody-
namic Properties
7

5.8 Thermal Expansion & Heat Transfer


8

5.9 Heat Capacity


 
Q = C T f − Ti (5.1)

FT
where C is the Heat Capacity and T f and Ti are the final and initial temperatures
respectively.

5.10 Specific Heat Capacity


 
Q = cm T f − ti (5.2)
RA
where c is the specific heat capacity and m is the mass.

5.11 Heat and Work


Z Vf
W= PdV (5.3)
Vi

5.12 First Law of Thermodynamics


D

dEint = dQ − dW (5.4)
where dEint is the internal energy of the system, dQ is the Energy added to the system
and dW is the work done by the system.

5.12.1 Special Cases to the First Law of Thermodynamics


Adiabatic Process During an adiabatic process, the system is insulated such that there
is no heat transfer between the system and its environment. Thus dQ = 0, so

∆Eint = −W (5.5)

If work is done on the system, negative W, then there is an increase in its internal
energy. Conversely, if work is done by the system, positive W, there is a decrease
in the internal energy of the system.

David S. Latchman ©2009


Work done by Ideal Gas at Constant Temperature 39
Constant Volume (Isochoric) Process If the volume is held constant, then the system
can do no work, δW = 0, thus
∆Eint = Q (5.6)
If heat is added to the system, the temperature increases. Conversely, if heat is
removed from the system the temperature decreases.

Closed Cycle In this situation, after certain interchanges of heat and work, the system
comes back to its initial state. So ∆Eint remains the same, thus

∆Q = ∆W (5.7)

The work done by the system is equal to the heat or energy put into it.

Free Expansion In this process, no work is done on or by the system. Thus ∆Q =


∆W = 0,

FT
∆Eint = 0 (5.8)

5.13 Work done by Ideal Gas at Constant Temperature


Starting with Equation 5.3, we substitute the Ideal gas Law, Equation 5.11, to get
Z Vf
dV
RA
W = nRT
Vi V
Vf
= nRT ln (5.9)
Vi

5.14 Heat Conduction Equation


The rate of heat transferred, H, is given by

Q TH − TC
H= = kA (5.10)
D

t L
where k is the thermal conductivity.

5.15 Ideal Gas Law


PV = nRT (5.11)
where

n = Number of moles
P = Pressure
V = Volume
T = Temperature

©2009 David S. Latchman


40 Thermodynamics & Statistical Mechanics
and R is the Universal Gas Constant, such that
R ≈ 8.314 J/mol. K
We can rewrite the Ideal gas Law to say
PV = NkT (5.12)
where k is the Boltzmann’s Constant, such that
R
k= ≈ 1.381 × 10−23 J/K
NA

5.16 Stefan-Boltzmann’s Formula


P(T) = σT4 (5.13)

5.17

5.18
RMS Speed of an Ideal Gas

Translational Kinetic Energy


FT
vrms =
r
3RT
M
(5.14)
RA
3
K̄ = kT (5.15)
2

5.19 Internal Energy of a Monatomic gas


3
Eint = nRT (5.16)
2

5.20 Molar Specific Heat at Constant Volume


D

Let us define, CV such that


Q = nCV ∆T (5.17)
Substituting into the First Law of Thermodynamics, we have
∆Eint + W = nCV ∆T (5.18)
At constant volume, W = 0, and we get
1 ∆Eint
CV = (5.19)
n ∆T
Substituting (Equation 5.16), we get
3
CV = R = 12.5 J/mol.K (5.20)
2

David S. Latchman ©2009


Molar Specific Heat at Constant Pressure 41
5.21 Molar Specific Heat at Constant Pressure
Starting with
Q = nCp ∆T (5.21)
and

∆Eint = Q − W
⇒ nCV ∆T = nCp ∆T + nR∆T
∴ CV = Cp − R (5.22)

5.22 Equipartition of Energy


!
f

FT
CV = R = 4.16 f J/mol.K (5.23)
2
where f is the number of degrees of freedom.

Table 5.1: Table of Molar Specific Heats

Degrees of Freedom Predicted Molar Specific Heats


Molecule Translational Rotational Total ( f ) CV CP = CV + R
RA
3 5
Monatomic 3 0 3 2
R 2
R
5 7
Diatomic 3 2 5 2
R 2
R
Polyatomic 3 3 6 3R 4R

5.23 Adiabatic Expansion of an Ideal Gas


PV γ = a constant (5.24)
D

where γ = CCVP .
We can also write
TV γ−1 = a constant (5.25)

5.24 Second Law of Thermodynamics


Something.

©2009 David S. Latchman


42 Thermodynamics & Statistical Mechanics

FT
RA
D

David S. Latchman ©2009


Chapter 6

Quantum Mechanics

6.1 Fundamental Concepts


1

6.2 Schrödinger Equation


Let us define Ψ to be FT
Ψ = Ae−iω(t− v )
x
(6.1)
RA
Simplifying in terms of Energy, E, and momentum, p, we get
i(Et−px)
Ψ = Ae− ~ (6.2)

We obtain Schrödinger’s Equation from the Hamiltonian

H =T+V (6.3)

To determine E and p,
D

∂2 Ψ p2
= − Ψ (6.4)
∂x2 ~2
∂Ψ iE
= Ψ (6.5)
∂t ~
and
p2
H= +V (6.6)
2m
This becomes

EΨ = HΨ (6.7)

~ ∂Ψ ∂2 Ψ
EΨ = − p2 Ψ = −~2
i ∂t ∂x2
44 Quantum Mechanics
The Time Dependent Schrödinger’s Equation is

∂Ψ ~ 2 ∂2 Ψ
i~ =− + V(x)Ψ (6.8)
∂t 2m ∂x2
The Time Independent Schrödinger’s Equation is

~ 2 ∂2 Ψ
EΨ = − + V(x)Ψ (6.9)
2m ∂x2

6.2.1 Infinite Square Wells


Let us consider a particle trapped in an infinite potential well of size a, such that

for 0 < x < a


(
0
V(x) =

FT
∞ for |x| > a,

so that a nonvanishing force acts only at ±a/2. An energy, E, is assigned to the system
such that the kinetic energy of the particle is E. Classically, any motion is forbidden
outside of the well because the infinite value of V exceeds any possible choice of E.
Recalling the Schrödinger Time Independent Equation, Equation 6.9, we substitute
V(x) and in the region (−a/2, a/2), we get

~2 d2 ψ
RA
− = Eψ (6.10)
2m dx2
This differential is of the form
d2 ψ
2
+ k2 ψ = 0 (6.11)
dx
where r
2mE
k= (6.12)
~2
We recognize that possible solutions will be of the form
D

cos kx and sin kx

As the particle is confined in the region 0 < x < a, we say

A cos kx + B sin kx for 0 < x < a


(
ψ(x) =
0 for |x| > a

We have known boundary conditions for our square well.

ψ(0) = ψ(a) = 0 (6.13)

It shows that

⇒ A cos 0 + B sin 0 = 0
∴A=0 (6.14)

David S. Latchman ©2009


Schrödinger Equation 45
We are now left with

B sin ka = 0
ka = 0; π; 2π; 3π; · · ·
(6.15)

While mathematically, n can be zero, that would mean there would be no wave function,
so we ignore this result and say

kn = for n = 1, 2, 3, · · ·
a
Substituting this result into Equation 6.12 gives

nπ 2mEn
kn = = (6.16)

FT
a ~
Solving for En gives
n2 π2 ~2
En = (6.17)
2ma2
We cna now solve for B by normalizing the function
Z a
a
|B|2 sin2 kxdx = |A|2 = 1
0 2
RA
2
So |A|2 = (6.18)
a
So we can write the wave function as
r
2 nπx
 
ψn (x) = sin (6.19)
a a

6.2.2 Harmonic Oscillators


Classically, the harmonic oscillator has a potential energy of
D

1
V(x) = kx2 (6.20)
2
So the force experienced by this particle is
dV
F=− = −kx (6.21)
dx
where k is the spring constant. The equation of motion can be summed us as

d2 x
m = −kx (6.22)
dt2
And the solution of this equation is
 
x(t) = A cos ω0 t + φ (6.23)

©2009 David S. Latchman


46 Quantum Mechanics
where the angular frequency, ω0 is
r
k
ω0 = (6.24)
m
The Quantum Mechanical description on the harmonic oscillator is based on the eigen-
function solutions of the time-independent Schrödinger’s equation. By taking V(x)
from Equation 6.20 we substitute into Equation 6.9 to get
d2 ψ 2m k 2
!
mk 2 2E
 
= x − E ψ = x − ψ
dx2 ~2 2 ~2 k
With some manipulation, we get
√
~ d2 ψ  mk 2 2E m 
r 
√ 2
=  x − ψ
mk dx ~ ~ k

FT
This step allows us to to keep some of constants out of the way, thus giving us

mk 2
ξ2 = x (6.25)
~r
2E m 2E
and λ = = (6.26)
~ k ~ω0
This leads to the more compact
RA
d2 ψ  2 
= ξ − λ ψ (6.27)
dξ2
where the eigenfunction ψ will be a function of ξ. λ assumes an eigenvalue anaglaous
to E.
From Equation 6.25, we see that the maximum value can be determined to be

mk 2
ξ2max = A (6.28)
~
Using the classical connection between A and E, allows us to say
D


mk 2E
ξmax =
2
=λ (6.29)
~ k
From Equation 6.27, we see that in a quantum mechanical oscillator, there are non-
vanishing solutions in the forbidden regions, unlike in our classical case.
A solution to Equation 6.27 is

ψ(ξ) = e−ξ /2
2
(6.30)
where

= −ξe−ξ /2
2


dψ 2 −xi2 /2 −ξ2 /2
e−ξ /2
  2
and 2
= ξ e − e = ξ 2
− 1

David S. Latchman ©2009


Schrödinger Equation 47
This gives is a special solution for λ where

λ0 = 1 (6.31)

Thus Equation 6.26 gives the energy eigenvalue to be

~ω0 ~ω0
E0 = λ0 = (6.32)
2 2

The eigenfunction e−ξ /2 corresponds to a normalized stationary-state wave function


2

! 18 √
mk mkx2 /2~ −iE0 t/~
Ψ0 (x, t) = 2 2 e− e (6.33)
π~

FT
This solution of Equation 6.27 produces the smallest possibel result of λ and E. Hence,
Ψ0 and E0 represents the ground state of the oscillator. and the quantity ~ω0 /2 is the
zero-point energy of the system.

6.2.3 Finite Square Well


For the Finite Square Well, we have a potential region where
RA
(
−V0 for −a ≤ x ≤ a
V(x) =
0 for |x| > a

We have three regions

Region I: x < −a In this region, The potential, V = 0, so Schrödinger’s Equation be-


comes

~2 d2 ψ
− = Eψ
D

2m dx2
d2 ψ
⇒ 2 = κ2 ψ
√ dx
−2mE
where κ=
~

This gives us solutions that are

ψ(x) = A exp(−κx) + B exp(κx)

As x → ∞, the exp(−κx) term goes to ∞; it blows up and is not a physically


realizable function. So we can drop it to get

ψ(x) = Beκx for x < −a (6.34)

©2009 David S. Latchman


48 Quantum Mechanics
Region II: −a < x < a In this region, our potential is V(x) = V0 . Substitutin this into
the Schrödinger’s Equation,Equation 6.9, gives
~2 d2 ψ
− − V0 ψ = Eψ
2m dx2
d2 ψ
or 2
= −l2 ψ
p dx
2m (E + V0 )
where l ≡ (6.35)
~
We notice that E > −V0 , making l real and positive. Thus our general solution
becomes
ψ(x) = C sin(lx) + D cos(lx) for −a < x < a (6.36)
Region III: x > a Again this Region is similar to Region III, where the potential, V = 0.

FT
This leaves us with the general solution
ψ(x) = F exp(−κx) + G exp(κx)
As x → ∞, the second term goes to infinity and we get
ψ(x) = Fe−κx for x > a (6.37)

This gives us  κx

 Be for x < a
ψ(x) =  for 0 < x < a

RA
D cos(lx) (6.38)

for x > a

 Fe−κx

6.2.4 Hydrogenic Atoms


c

6.3 Spin
3
D

6.4 Angular Momentum


4

6.5 Wave Funtion Symmetry


5

6.6 Elementary Perturbation Theory


6

David S. Latchman ©2009


Chapter 7

Atomic Physics

7.1 Properties of Electrons


1

7.2 Bohr Model


FT
To understand the Bohr Model of the Hydrogen atom, we will take advantage of our
knowlegde of the wavelike properties of matter. As we are building on a classical
RA
model of the atom with a modern concept of matter, our derivation is considered to be
‘semi-classical’. In this model we have an electron of mass, me , and charge, −e, orbiting
a proton. The cetripetal force is equal to the Coulomb Force. Thus

1 e2 me v2
= (7.1)
4π0 r2 r

The Total Energy is the sum of the potential and kinetic energies, so

p2
E=K+U = − | f race2 4π0 r (7.2)
D

2me

We can further reduce this equation by subsituting the value of momentum, which we
find to be
p2 1 e2
= me v2 = (7.3)
2me 2 8π0 r
Substituting this into Equation 7.2, we get

e2 e2 e2
E= − =− (7.4)
8π0 r 4π0 r 8π0 r

At this point our classical description must end. An accelerated charged particle, like
one moving in circular motion, radiates energy. So our atome here will radiate energy
and our electron will spiral into the nucleus and disappear. To solve this conundrum,
Bohr made two assumptions.
50 Atomic Physics
1. The classical circular orbits are replaced by stationary states. These stationary
states take discreet values.

2. The energy of these stationary states are determined by their angular momentum
which must take on quantized values of ~.

L = n~ (7.5)

We can find the angular momentum of a circular orbit.

L = m3 vr (7.6)

From Equation 7.1 we find v and by substitution, we find L.

FT
r
m3 r
L=e (7.7)
4π0

Solving for r, gives


L2
r= (7.8)
me e2 /4π0
RA
We apply the condition from Equation 7.5

n2 ~2
rn = = n2 a0 (7.9)
me e2 /4π0

where a0 is the Bohr radius.


a0 = 0.53 × 10−10 m (7.10)

Having discreet values for the allowed radii means that we will also have discreet
values for energy. Replacing our value of rn into Equation 7.4, we get
D

!
me e2 13.6
En = − 2 = − 2 eV (7.11)
2n 4π0 ~ n

7.3 Energy Quantization


3

7.4 Atomic Structure


4

David S. Latchman ©2009


Atomic Spectra 51
7.5 Atomic Spectra
7.5.1 Rydberg’s Equation
1 1 1
 
= RH 02 − 2 (7.12)
λ n n
where RH is the Rydberg constant.
For the Balmer Series, n0 = 2, which determines the optical wavelengths. For
n0 = 3, we get the infrared or Paschen series. The fundamental n0 = 1 series falls in the
ultraviolet region and is known as the Lyman series.

7.6 Selection Rules

FT
6

7.7 Black Body Radiation


7.7.1 Plank Formula
8π~ f3
u( f, T) = (7.13)
c3 eh f /kT − 1
RA
7.7.2 Stefan-Boltzmann Formula
P(T) = σT4 (7.14)

7.7.3 Wein’s Displacement Law


λmax T = 2.9 × 10−3 m.K (7.15)

7.7.4 Classical and Quantum Aspects of the Plank Equation


D

Rayleigh’s Equation
8π f 2
u( f, T) = kT (7.16)
c3
We can get this equation from Plank’s Equation, Equation 7.13. This equation is a
classical one and does not contain Plank’s constant in it. For this case we will look at
the situation where h f < kT. In this case, we make the approximation

ex ' 1 + x (7.17)

Thus the demonimator in Equation 7.13 becomes

hf hf
eh f /kT − 1 ' 1 + −1= (7.18)
kT kT

©2009 David S. Latchman


52 Atomic Physics
Thus Equation 7.13 takes the approximate form

8πh 3 kT 8π f 2
u( f, T) ' f = 3 kT (7.19)
c3 hf c

As we can see this equation is devoid of Plank’s constant and thus independent of
quantum effects.

Quantum
At large frequencies, where h f > kT, quantum effects become apparent. We can
estimate that
eh f /kT − 1 ' eh f /kT (7.20)
Thus Equation 7.13 becomes

7.8
7.8.1
X-Rays
Bragg Condition
u( f, T) '

FT8πh 3 −h f /kT
c3
f e (7.21)
RA
2d sin θ = mλ (7.22)
for constructive interference off parallel planes of a crystal with lattics spacing, d.

7.8.2 The Compton Effect


The Compton Effect deals with the scattering of monochromatic X-Rays by atomic
targets and the observation that the wavelength of the scattered X-ray is greater than
the incident radiation. The photon energy is given by
D

hc
E = hυ = (7.23)
λ
The photon has an associated momentum

E= pc (7.24)
E hυ h
⇒p = = = (7.25)
c c λ
The Relativistic Energy for the electron is

E2 = p2 c2 + m2e c4 (7.26)

where
p − p0 = P (7.27)

David S. Latchman ©2009


Atoms in Electric and Magnetic Fields 53
Squaring (Equation 7.27) gives

p2 − 2p · p0 + p02 = P2 (7.28)

Recall that E = pc and E 0 = cp0 , we have

c2 p2 − 2c2 p · p0 + c2 p02 = c2 P2
E 2 − 2E E 0 cos θ + E 02 = E2 − m2e c4 (7.29)

Conservation of Energy leads to

E + me c2 = E 0 + E (7.30)

Solving

E − E 0 = E − me c2

FT
E 2 − 2E E 0 + E 0 = E2 − 2Eme c2 + m2e c4 (7.31)
2E E 0 − 2E E 0 cos θ = 2Eme c2 − 2m2e c4 (7.32)

Solving leads to
h
∆λ = λ0 − λ = (1 − cos θ) (7.33)
me c
where λc = h
me c
is the Compton Wavelength.
RA
h
λc = = 2.427 × 10−12 m (7.34)
me c

7.9 Atoms in Electric and Magnetic Fields


7.9.1 The Cyclotron Frequency
A test charge, q, with velocity v enters a uniform magnetic field, B. The force acting on
the charge will be perpendicular to v such that
D

FB = qv × B (7.35)

or more simply FB = qvB. As this traces a circular path, from Newton’s Second Law,
we see that
mv2
FB = = qvB (7.36)
R
Solving for R, we get
mv
R= (7.37)
qB
We also see that
qB
f = (7.38)
2πm
The frequency is depends on the charge, q, the magnetic field strength, B and the mass
of the charged particle, m.

©2009 David S. Latchman


54 Atomic Physics
7.9.2 Zeeman Effect
The Zeeman effect was the splitting of spectral lines in a static magnetic field. This is
similar to the Stark Effect which was the splitting in the presence in a magnetic field.
In the Zeeman experiment, a sodium flame was placed in a magnetic field and its
spectrum observed. In the presence of the field, a spectral line of frequency, υ0 was
split into three components, υ0 − δυ, υ0 and υ0 + δυ. A classical analysis of this effect
allows for the identification of the basic parameters of the interacting system.
The application of a constant magnetic field, B, allows for a direction in space in
which the electron motion can be referred. The motion of an electron can be attributed
to a simple harmonic motion under a binding force −kr, where the frequency is
r
1 k
υ0 = (7.39)
2π me

FT
The magnetic field subjects the electron to an additional Lorentz Force, −ev × B. This
produces two different values for the angular velocity.
v = 2πrυ
The cetripetal force becomes
me v2
= 4π2 υ2 rme
r
Thus the certipetal force is
RA
4π2 υ2 rme = 2πυreB + kr for clockwise motion
4π2 υ2 rme = −2πυreB + kr for counterclockwise motion
We use Equation 7.39, to emiminate k, to get
eB
υ2 − υ − υ0 = 0 (Clockwise)
2πme
eB
υ2 + υ − υ0 = 0 (Counterclockwise)
2πme
D

As we have assumed a small Lorentz force, we can say that the linear terms in υ are
small comapred to υ0 . Solving the above quadratic equations leads to
eB
υ = υ0 + for clockwise motion (7.40)
4πme
eB
υ = υ0 − for counterclockwise motion (7.41)
4πme
We note that the frequency shift is of the form
eB
δυ = (7.42)
4πme
If we view the source along the direction of B, we will observe the light to have two
polarizations, a closckwise circular polarization of υ0 + δυ and a counterclosckwise
circular polarization of υ0 − δυ.

David S. Latchman ©2009


Atoms in Electric and Magnetic Fields 55
7.9.3 Franck-Hertz Experiment
The Franck-Hertz experiment, performed in 1914 by J. Franck and G. L. Hertz, mea-
sured the colisional excitation of atoms. Their experiement studied the current of
electrons in a tub of mercury vapour which revealed an abrupt change in the current
at certain critical values of the applied voltage.1 They interpreted this observation as
evidence of a threshold for inelastic scattering in the colissions of electrons in mer-
cury atoms.The bahavior of the current was an indication that electrons could lose
a discreet amount of energy and excite mercury atoms in their passage through the
mercury vapour. These observations constituted a direct and decisive confirmation of
the existence os quantized energy levels in atoms.

FT
RA
D

1
Put drawing of Franck-Hertz Setup

©2009 David S. Latchman


56 Atomic Physics

FT
RA
D

David S. Latchman ©2009


Chapter 8

Special Relativity

8.1 Introductory Concepts


8.1.1 Postulates of Special Relativity

FT
1. The laws of Physics are the same in all inertial frames.

2. The speed of light is the same in all inertial frames.

We can define
RA
1
γ= q (8.1)
u2
1− c2

8.2 Time Dilation


∆t = γ∆t0 (8.2)
where ∆t0 is the time measured at rest relative to the observer, ∆t is the time measured
in motion relative to the observer.
D

8.3 Length Contraction


L0
L= (8.3)
γ
where L0 is the length of an object observed at rest relative to the observer and L is the
length of the object moving at a speed u relative to the observer.

8.4 Simultaneity
4
58 Special Relativity
8.5 Energy and Momentum
8.5.1 Relativistic Momentum & Energy
In relativistic mechanics, to be conserved, momentum and energy are defined as

Relativistic Momentum
p̄ = γmv̄ (8.4)

Relativistic Energy
E = γmc2 (8.5)

FT
8.5.2 Lorentz Transformations (Momentum & Energy)

E
 
p0x = γ px − β (8.6)
c
py = py
0
(8.7)
p0z = pz (8.8)
RA
E0 E
 
=γ − βpx (8.9)
c c

8.5.3 Relativistic Kinetic Energy

K = E − mc2 (8.10)
 
 
 1 
= mc  q
2
− 1 (8.11)
D

v2
1−
 
c2

= mc2 γ − 1

(8.12)

8.5.4 Relativistic Dynamics (Collisions)

∆E
 
∆P0x = γ ∆Px − β (8.13)
c
∆P y = ∆P y
0
(8.14)
∆P0z = ∆Pz (8.15)
∆E0 ∆E
 
=γ − β∆Px (8.16)
c c

David S. Latchman ©2009


Four-Vectors and Lorentz Transformation 59
8.6 Four-Vectors and Lorentz Transformation
We can represent an event in S with the column matrix, s,

 x 
 
 y 
s =  (8.17)
 
 z 

 
ict

A different Lorents frame, S0 , corresponds to another set of space time axes so that
 0 
 x 
 y0 
s0 =  0  (8.18)
 
 z 
 0 
ict

FT
The Lorentz Transformation is related by the matrix

 x   γ
 0  
0 0 iγβ   x 
 
 y0   0 1 0 0   y 
 0  =  (8.19)
  
 z   0 0 1 0   z 
 
0 γ
 0    
ict −iγβ 0 ict

We can express the equation in the form


RA
s0 = L s (8.20)

The matrix L contains all the information needed to relate position four–vectors for
any given event as observed in the two Lorentz frames S and S0 . If we evaluate

 x 
 
h i  y 
s s = x y z ict 
T  = x2 + y2 + z2 − c2 t2 (8.21)
 z 
 
ict
D

Similarly we can show that

s0T s0 = x02 + y02 + z02 − c2 t02 (8.22)

We can take any collection of four physical quantities to be four vector provided that
they transform to another Lorentz frame. Thus we have

 bx 
 
 b 
b =  y  (8.23)
 
 bz 
 
ibt

this can be transformed into a set of quantities of b0 in another frame S0 such that it
satisfies the transformation
b0 = L b (8.24)

©2009 David S. Latchman


60 Special Relativity
Looking at the momentum-Energy four vector, we have

 px
 

 p 
p =  y (8.25)
 
 pz


 
iE/c

Applying the same transformation rule, we have

p0 = L p (8.26)

We can also get a Lorentz-invariation relation between momentum and energy such
that
p0T p0 = pT p (8.27)

FT
The resulting equality gives

E02 E2
x + p y + pz −
p02 = + +
02 02 2 2 2
px p y p z − (8.28)
c2 c2

8.7 Velocity Addition


v−u
v0 =
RA
(8.29)
1 − uv
c2

8.8 Relativistic Doppler Formula


r r
c+u c−u
ῡ = υ0 let r = (8.30)
c−u c+u
We have

ῡreceding = rυ0 red-shift (Source Receding) (8.31)


D

υ0
ῡapproaching = blue-shift (Source Approaching) (8.32)
r

8.9 Lorentz Transformations


Given two reference frames S(x, y, z, t) and S0 (x0 , y0 , z0 , t0 ), where the S0 -frame is moving
in the x-direction, we have,

x0 = γ (x − ut) x = (x0 − ut0 ) (8.33)


y0 = y y = y0 (8.34)
z0 = y y0 = y (8.35)
u u 0
   
t = γ t − 2x
0
t = γ t + 2x
0
(8.36)
c c

David S. Latchman ©2009


Space-Time Interval 61
8.10 Space-Time Interval
(∆S)2 = (∆x)2 + ∆y 2 + (∆z)2 − c2 (∆t)2

(8.37)
Space-Time Intervals may be categorized into three types depending on their separa-
tion. They are

Time-like Interval

c2 ∆t2 > ∆r2 (8.38)


∆S2 > 0 (8.39)

When two events are separated by a time-like interval, there is a cause-effect


relationship between the two events.

FT
Light-like Interval

c2 ∆t2 = ∆r2 (8.40)


S =0
2
(8.41)

Space-like Intervals

c2 ∆t2 < ∆r2 (8.42)


RA
∆S < 0 (8.43)
D

©2009 David S. Latchman


62 Special Relativity

FT
RA
D

David S. Latchman ©2009


Chapter 9

Laboratory Methods

9.1 Data and Error Analysis


9.1.1 Addition and Subtraction

The Error in x is
FT
x=a+b−c

(δx)2 = (δa)2 + (δb)2 + (δc)2


(9.1)

(9.2)
RA
9.1.2 Multiplication and Division
a×b
x= (9.3)
c
The error in x is
!2
δx δa δb δc
 2  2  2
= + + (9.4)
x a b c
D

9.1.3 Exponent - (No Error in b)

x = ab (9.5)

The Error in x is
δx δa
 
=b (9.6)
x a

9.1.4 Logarithms
Base e

x = ln a (9.7)
64 Laboratory Methods
We find the error in x by taking the derivative on both sides, so
d ln a
δx = · δa
da
1
= · δa
a
δa
= (9.8)
a

Base 10
x = log10 a (9.9)
The Error in x can be derived as such
d(log a)
δx = δa
da

FT
ln a
ln 10
= δa
da
1 δa
=
ln 10 a
δa
= 0.434 (9.10)
a

9.1.5 Antilogs
RA
Base e
x = ea (9.11)
We take the natural log on both sides.
ln x = a ln e = a (9.12)
Applaying the same general method, we see
d ln x
δx = δa
dx
D

δx
⇒ = δa (9.13)
x

Base 10
x = 10a (9.14)
We follow the same general procedure as above to get
log x = a log 10
log x
δx = δa
dx
1 d ln a
δx = δa
ln 10 dx
δx
= ln 10δa (9.15)
x

David S. Latchman ©2009


Instrumentation 65
9.2 Instrumentation
2

9.3 Radiation Detection


3

9.4 Counting Statistics


Let’s assume that for a particular experiment, we are making countung measurements
for a radioactive source. In this experiment, we recored N counts in time T. The
counting rate for this trial is R = N/T. This rate should be close to the average√rate, R̄.

FT
The standard deviation or the uncertainty of our count is a simply called the N rule.
So √
σ= N (9.16)
Thus we can report our results as

Number of counts = N ± N (9.17)

We can find the count rate by dividing by T, so


RA

N N
R= ± (9.18)
T T
δN
The fractional uncertainty of our count is N
. We can relate this in terms of the count
rate.
δN
δR T δN
= N
=
R T
N

N
D

=
N
1
= (9.19)
N
We see that our uncertainty decreases as we take more counts, as to be expected.

9.5 Interaction of Charged Particles with Matter


5

9.6 Lasers and Optical Interferometers


6

©2009 David S. Latchman


66 Laboratory Methods
9.7 Dimensional Analysis
Dimensional Analysis is used to understand physical situations involving a mis of
different types of physical quantities. The dimensions of a physical quantity are
associated with combinations of mass, length, time, electric charge, and temperature,
represented by symbols M, L, T, Q, and θ, respectively, each raised to rational powers.

9.8 Fundamental Applications of Probability and Statis-


tics
8

FT
RA
D

David S. Latchman ©2009


Chapter 10

GR8677 Exam Solutions

10.1 Motion of Rock under Drag Force

rock.

FT
From the information provided we can come up with an equation of motion for the

mẍ = −mg − kv (10.1)

If you have seen this type of equation, and solved it, you’d know that the rock’s speed
will asymtotically increase to some max speed. At that point the drag force and the
force due to gravity will be the same. We can best answer this question through analysis
RA
and elimination.

A Dividing Equation 10.1 by m gives

k
ẍ = −g − ẋ (10.2)
m

We see that this only occurs when ẋ = 0, which only happens at the top of the
flight. So FALSE.
D

B From Equation 10.2, we see that this is TRUE.

C Again from Equation 10.2 we see that the acceleration is dependent on whether the
rock is moving up or down. If ẋ > 0 then ẍ < −g and if ẋ < 0 then ẍ > −g. So this
is also FALSE.

D If there was no drag (fictional) force, then energy would be conserved and the rock
will return at the speed it started with but there is a drag force so energy is lost.
The speed the rock returns is v < v0 . Hence FALSE.

E Again FALSE. Once the drag force and the gravitational force acting on the rock is
balanced the rock won’t accelerate.

Answer: (B)
68 GR8677 Exam Solutions
10.2 Satellite Orbits
The question states that the astronaut fires the rocket’s jets towards Earth’s center. We
infer that no extra energy is given to the system by this process. subsection 2.7.5, shows
that the only other orbit where the specific energy is also negative is an elliptical one.
Answer: (A)

10.3 Speed of Light in a Dielectric Medium


Solutions to the Electromagnetic wave equation gives us the speed of light in terms of
the electromagnetic permeability, µ0 and permitivitty, 0 .

1
c= √ (10.3)

FT
µ0 0

where c is the speed of light. The speed through a dielectric medium becomes

1
v = √
µ0
1
=
2.1µ0 0
p
RA
c
= √ (10.4)
2.1
Answer: (D)

10.4 Wave Equation


We are given the equation
t x
 
y = A sin − (10.5)
T λ
D

We can analyze and eliminate from what we know about this equation

A The Amplitude, A in the equation is the displacement from equilibrium. So this


choice is incorrect.
 
B As the wave moves, we seek to keep the Tt − λx term constant. So as t increases, we
expect x to increase as well as there is a negative sign in front of it. This means
that the wave moves in the positive x-direction. This choice is also incorrect.
 
C The phase of the wave is given by Tt − λx , we can do some manipulation to show

t x
 
− = 2π f t − kx
T λ
= ωt − kx (10.6)

David S. Latchman ©2009


Inelastic Collision and Putty Spheres 69
Or rather
kx = ωt (10.7)

Differentiating Equation 10.7 gives us the phase speed, which is

λ
v= (10.8)
T

This is also incorrect

E From Equation 10.8 the above we see that is the answer.

Answer: (E)

10.5

FT
Inelastic Collision and Putty Spheres
We are told the two masses coalesce so we know that the collision is inelastic and
hence, energy is not conserved. As mass A falls, it looses Potential Energy and gains
Kinetic Energy.
1
Mgh0 = Mv20 (10.9)
RA
2
Thus
v20 = 2gh0 (10.10)

Upon collision, momentum is conserved, thus

Mv0 = (3M + M) v1
= 4Mv1
v0
⇒ v1 = (10.11)
4
D

The fused putty mass rises, kinetic energy is converted to potential energy and we find
our final height.

1
(4M) v21 = 4Mgh
2
v21
h =
2g
1 v0 2
 
=
2g 4
h0
= (10.12)
16

Answer: (A)

©2009 David S. Latchman


70 GR8677 Exam Solutions
10.6 Motion of a Particle along a Track
As the particle moves from the top of the track and runs down the frictionless track,
its Gravitational Potential Energy is converted to Kinetic Energy. Let’s assume that the
particle is at a height, y0 when x = 0. Since energy is conserved, we get1

1
mgy0 = mg(y0 − y) + mv2
2
1 2
⇒ v = gy (10.13)
2
So we have a relationship between v and the particle’s position on the track.
The tangential acceleration in this case is

mv2

FT
mg cos θ = (10.14)
r

where r is the radius of curvature and is equal to x2 + y2 .


p

Substituting this into Equation 10.14 gives

v2
g cos θ =
r
gx2
=
RA
2 x2 + y2
p
gx
= √ (10.15)
x2 + 4
Answer: (D)

10.7 Resolving Force Components


This question is a simple matter of resolving the horizontal and vertical components
D

of the tension along the rope. We have

T sin θ = F (10.16)
T cos θ = mg (10.17)

Thus we get

F
tan θ =
mg
10 1
= ≈ (10.18)
(2)(9.8) 2

Answer: (A)
1
Insert Free Body Diagram of particle along track.

David S. Latchman ©2009


Nail being driven into a block of wood 71
10.8 Nail being driven into a block of wood
We recall that
v2 = v20 + 2as (10.19)

where v, v0 , a and s are the final speed, initial speed, acceleration and displacement
that the nail travels. Now it’s just a matter os plugging in what we know

0 = 100 + 2a(0.025) (10.20)


100
⇒a = − = −2000 m/s2 (10.21)
2(0.025)

The Force on the nail comes from Newton’s Second Law

FT
F = ma
= 5 · 2000 = 10000 N (10.22)

Answer: (D)

10.9 Current Density


RA
We can find the drift vleocity from the current density equation

J = envd (10.23)

where e is the charge of an electron, n is the density of electrons per unit volume and
~
vd is the drift speed. Plugging in what we know

I
J=
A
D

I =nAvd e
I
vd =
nAe
100
= (10.24)
(1 × 1028 ) π×2×10
−4
4
1.6 × 10−19

paying attention to the indices of the equation we get

2 − 28 + 4 + 19 = −4 (10.25)

So we expect an answer where vd ≈ 10−4 .2


Answer: (D)
2
It also helps if you knew that the electron drift velocity was slow, in the order of mm/s.

©2009 David S. Latchman


72 GR8677 Exam Solutions
10.10 Charge inside an Isolated Sphere
You can answer this by thinking about Gauss’ Law. The bigger the Gaussian surface
the more charge it encloses and the bigger the electric field. Beyond the radius of the
sphere, the field decreases exponentially.
We can calculate these relationships by using Gauss’ Law.
I
Qenclosed
E · dS = (10.26)
0
S

where the current density, ρ is

Q Qenclosed
ρ= = (10.27)

FT
4
3
πR3 4
3
πr3

where R is the radius of the sphere.

for r < R The enclosed charge becomes

4 3 Qr3
 
Qenclosed = ρ πr = 3 (10.28)
3 R
RA
Gauss’ Law becomes
  Qr3
E 4πr2 = (10.29)
0 R3
The Electric field is
Qr
E(r<R) = (10.30)
4π0 R3

This is a linear relationship with respect to r.


D

for r ≥ R The enclosed charge is


Qenclosed = Q (10.31)

Gauss’ Law becomes


  Q
E 4πr2 = (10.32)
0
The Electric field is
Q
E(r≥R) = (10.33)
4π0 r2

The linear increase is exhibited by choice (C).


Answer: (C)

David S. Latchman ©2009


Vector Identities and Maxwell’s Laws 73
10.11 Vector Identities and Maxwell’s Laws
We recall the vector identity
∇ · (∇ × A) = 0 (10.34)
Thus
 
∇ · (∇ × H) = ∇ · Ḋ + J
= 0 (10.35)

Answer: (A)

10.12 Doppler Equation (Non-Relativistic)

FT
we recall the Doppler Equation3
v − vr
 
f = f0 (10.36)
v − vs
where vr and vs are the observer and source speeds respectively. We are told that vr = 0
and vs = 0.9v. Thus
v
 
f = f0
RA
v − 0.9v
= 10 f0
= 10 kHz (10.37)

Answer: (E)

10.13 Vibrating Interference Pattern


Answering this question takes some analysis. The sources are coherent, so they will
produce an interference pattern. We are also told that ∆ f = 500 Hz. This will produce
D

a shifting interference pattern that changes too fast for the eye to see.4
Answer: (E)

10.14 Specific Heat at Constant Pressure and Volume


From section 5.20 and section 5.21, we see that

Cp = CV + R (10.38)

The difference is due to the work done in the environment by the gas when it expands
under constant pressure.
3
Add reference to Dopler Equations.
4
Add Young’s Double Slit Experiment equations.

©2009 David S. Latchman


74 GR8677 Exam Solutions
We can prove this by starting with the First Law of Thermodynamics.

dU = −dW + dQ (10.39)

Where dU is the change in Internal Energy, dW is the work done by the system and dQ
is the change in heat of the system.
We also recall the definition for Heat Capacity

dQ = CdT (10.40)

At constant volume, there is no work done by the system, dV = 0. So it follows that


dW = 0. The change in internal energy is the change of heat into the system, thus we
can define, the heat capacity at constant volume to be

dUV = CV dT = dQV (10.41)

FT
At constant pressure, the change in internal energy is accompanied by a change in heat
flow as well as a change in the volume of the gas, thus

dUp = −dWp + dQp


= −pdV + Cp dT where pdV = nRdT
= −nRdT + Cp dT (10.42)

If the changes in internal energies are the same in both cases, then Equation 10.42 is
RA
equal to Equation 10.41. Thus

CV dT = −nRdT + Cp dT

This becomes
Cp = CV + nR (10.43)
We see the reason why Cp > CV is due to the addition of work on the system; Equa-
tion 10.41 shows no work done by the gas while Equation 10.42 shows that there is
work done.
Answer: (A)
D

10.15 Helium atoms in a box


Let’s say the probability of the atoms being inside the box is 1. So the probability that
an atom will be found outside of a 1.0 × 10−6 cm3 box is

P = 1 − 1.0 × 10−6 (10.44)

As there are N atoms and the probability of finding one is mutually exclusive of the
other, the probabolity becomes
 N
P = 1 − 1.0 × 10−6 (10.45)

Answer: (C)

David S. Latchman ©2009


The Muon 75
10.16 The Muon
It helps knowing what these particles are
Muon The muon, is a lepton, like the electron. It has the ame charge, −e and spin, 1/2,
as the electron execpt it’s about 200 times heavier. It’s known as a heavy electron.

Electron This is the answer.

Graviton This is a hypothetical particle that mediates the force of gravity. It has no
charge, no mass and a spin of 2. Nothing like an electron.

Photon The photon is the quantum of the electromagnetic field. It has no charge or
mass and a spin of 1. Again nothing like an electron.

Pion The Pion belongs to the meson family. Again, nothing like leptons.

FT
Proton This ia a sub atomic particle and is found in the nucleus of all atoms. Nothing
like an electron.
Answer: (A)

10.17 Radioactive Decay


RA
From the changes in the Mass and Atomic numbers after the subsequent decays, we
expect an α and β decay.
Alpha Decay
Z−2 X +2 α
A
ZX →A−4 0 4
(10.46)

Beta Decay
A
ZX →AZ+1 X0 +−1 e− + ῡe (10.47)

Combining both gives


D

A
ZX
A−4
→Z−2 X0 +42 α →AZ−1 Y +−1 e− + ῡe (10.48)

Answer: (B)

10.18 Schrödinger’s Equation


We recall that Schrödinger’s Equation is

~2 ∂2 ψ
Eψ = − + V(x)ψ (10.49)
2m ∂x2

Given that ( 2 2)
bx
ψ(x) = A exp − (10.50)
2

©2009 David S. Latchman


76 GR8677 Exam Solutions
We differentiate and get
∂2 ψ  4 2 
= b x − b 2
ψ (10.51)
∂x2
Plugging into Schrödinger’s Equation, Equation 10.49, gives us

~2  4 2 
Eψ = − b x − b2 ψ + V(x)ψ (10.52)
2m
Applying the boundary condition at x = 0 gives

~2 2
Eψ = − bψ (10.53)
2m
This gives
~2 b2 ~2  4 2 
− ψ=− b x − b2 ψ + v(x)ψ (10.54)

FT
2m 2m
Solving for V(x) gives
~2 b4 x2
V(x) = (10.55)
2m
Answer: (B)

10.19 Energy Levels of Bohr’s Hydrogen Atom


RA
We recall that the Energy Levels for the Hydrogen atom is

Z2
En = − 13.6 eV (10.56)
n2

where Z is the atomic number and n is the quantum number. This can easily be reduced
to
A
En = − 2 (10.57)
n
D

Answer: (E)

10.20 Relativistic Energy


The Rest Energy of a particle is given

E = mc2 (10.58)

The Relativistic Energy is for a relativistic particle moving at speed v

E = γv mc2 (10.59)

We are told that a kaon moving at relativistic speeds has the same energy as the rest
mass as a proton. Thus
EK + = Ep (10.60)

David S. Latchman ©2009


Space-Time Interval 77
where

EK+ = γv mK+ c2 (10.61)


Ep = mp c2 (10.62)

Equating both together gives


mp
γv = (10.63)
mK+
939
= (10.64)
494
940
≈ 500
(10.65)

This becomes

γv ≈ 1.9

FT
(10.66)

Solving gives
2.61 2
v2 = c (10.67)
3.61
This gives v2 in the order of 0.7. Squaring will give an answer that’s greater than 0.7,
the only answer is 0.85c.
Answer: (E)
RA
10.21 Space-Time Interval
We recall the Space-Time Interval from section 8.10.

(∆S)2 = (∆x)2 + ∆y 2 + (∆z)2 − c2 (∆t)2



(10.68)

We get

∆S2 = (5 − 3)2 + (3 − 3)2 + (3 − 1)2 − c2 (5 − 3)2


= 22 + 02 + 22 − 22
D

= 22
∆S = 2 (10.69)

Answer: (C)

10.22 Lorentz Transformation of the EM field


Lorentz transformations show that electric and magnetic fields are different aspects of
the same force; the electromagnetic force. If there was one stationary charge in our
rest frame, we would observe an electric field. If we were to move to a moving frame
of reference, Lorentz transformations predicts the presence of an additional magnetic
field.
Answer: (B)

©2009 David S. Latchman


78 GR8677 Exam Solutions
10.23 Conductivity of a Metal and Semi-Conductor
More of a test of what you know.

A Copper is a conductor so we expect its conductivity to be much greater than that of


a semiconductor. TRUE.

B As the temperature of the conductor is increased its atoms vibrate more and disrupt
the flow of electrons. As a result, resistance increases. TRUE.

C Different process. As temperature increases, electrons gain more energy to jump the
energy barrier into the conducting region. So conductivity increases. TRUE.

D You may have paused to think for this one but this is TRUE. The addition of an
impurity causes an increase of electron scattering off the impurity atoms. As a

FT
result, resistance increases.5

E The effect of adding an impurity on a semiconductor’s conductivity depends on


how many extra valence electrons it adds or subtracts; you can either widen or
narrow the energy bandgap. This is of crucial importance to electronics today.
So this is FALSE.

Answer: (E)
RA
10.24 Charging a Battery
The Potential Difference across the resistor, R is

PD = 120 − 100 = 20 V (10.70)

The Total Resistance is


V
R+r=
I
D

20
=
10
R+1=2
⇒ R = 1Ω (10.71)

Answer: (C)

10.25 Lorentz Force on a Charged Particle


We are told that the charged particle is released from rest in the electric and magnetic
fields. The charged particle will experience a force from the magnetic field only when
5
There are one or two cases where this is not true. The addition of Silver increases the conductivity
of Copper. But the conductivity will still be less than pure silver.

David S. Latchman ©2009


K-Series X-Rays 79
it moves perpendicular to the direction of the magnetic field lines. The particle will
move along the direction of the electric field.
We can also anylize this by looking at the Lorentz Force equation

Fq = q [E + (v × B)] (10.72)

v is in the same direction as B so the cross product between them is zero. We are left
with
Fq = qE (10.73)
The force is directed along the electrical field line and hence it moves in a straight line.
Answer: (E)

10.26 K-Series X-Rays

eff n

 1
FT
To calculate we look at the energy levels for the Bohr atom. As the Bohr atom considers
the energy levels for the Hydrogen atom, we need to modify it somewhat

f
1
ni


En = Z2  2 − 2  13.6 eV (10.74)
RA
where Zeff is the effective atomic number and n f and ni are the energy levels. For the
n f = 1 transition
Zeff = Z − 1 (10.75)
where Z = 28 for nickle. As the electrons come in from ni = ∞, Equation 10.74 becomes

1 1
 
E1 = (28 − 1)2 − 13.6 eV (10.76)
1 2 ∞2
This works out to

E1 = (272 )13.6 eV
D

≈ (30)2 × 13.6 eV (10.77)

This takes us in the keV range.


Answer: (D)

10.27 Electrons and Spin


It helps if you knew some facts here.

A The periodic table’s arrangement of elements tells us about the chemical properties
of an element and these properties are dependent on the valent electrons. How
these valent electrons are arranged is, of course, dependent on spin. So this
choice is TRUE.

©2009 David S. Latchman


80 GR8677 Exam Solutions
B The energy of an elecron is quantized and obey the Pauli’s Exclusion Principle. All
the electrons are accomodated from the lowest state up to the Fermi Level and the
distribution among levels is described by the Fermi distribution function, f (E),
which defines the probability that the energy level, E, is occupied by an electron.

E < EF
(
1,
f (E) =
0, E > EF

where f (E) is the Fermi-Dirac Distribution

1
f (E) = (10.78)
eE−EF /kT + 1

As a system goes above 0K, thermal energy may excite to higher energy states

FT
but this energy is not shared equally by all the electrons. The way energy is
distributed comes about from the exclusion principle, the energy an electron my
absorb at room temperture is kT which is much smaller than EF = 5eV. We can
define a Fermi Temperature,
EF = kTF (10.79)

which works out to be, TF = 60000K. Thus only electrons close to this temperature
can be excited as the levels above EF are empty. This results in a small number
RA
of electrons being able to be thermally excited and the low electronic specific
heat.[1]
π2
!
T
C= Nk where kT << EF
2 Tf

So this choice is also TRUE.

C The Zeeman Effect describes what happens to Hydrogen spectral lines in a magnetic
field; the spectral lines split. In some atoms, there were further splits in spctral
lines that couln’t be explained by magnetic dipole moments. The explanation for
D

this additional splitting was discovered to be due to electron spin.6

D The deflection of an electron in a uniform magnetic field deflects only in one way
and demonstrates none of the electron’s spin properties. Electrons can be de-
flected depending on their spin if placed in a non-uniform magnetic field, as was
demonstrated in the Stern-Gerlach Experiment.7

E When the Hydrogen spectrum is observed at a very high resolution, closely spaced
doublets are observed. This was one of the first experimental evidence for electron
spin.8

6
Write up on Zeeman and anomalous Zeemen effects
7
Write up on Stern-Gerlach Experiment
8
Write up on Fine Structure

David S. Latchman ©2009


Normalizing a wavefunction 81
10.28 Normalizing a wavefunction
We are given
ψ(φ) = Aeimφ (10.80)
Normalizing a function means
Z ∞
|Ψ(x)|2 dx = 1 (10.81)
−∞

In this case, we want Z 2π 2


ψ(φ) dφ = 1 (10.82)
0

and that 2

FT

ψ(x) = ψ∗ (x)ψ(x) (10.83)
So
2
⇒ ψ(φ) = A2 eimφ e−imφ
Z 2π
A 2
dφ = 1
0
A [2π − 0] = 1
2
RA
1
⇒A = √ (10.84)

10.29 Right Hand Rule


First we use the ‘Grip’ rule to tell what direction the magnetic field lines are going.
Assuming the wire and current are coming out of the page, the magnetic field is in a
clockwise direction around the wire. Now we can turn to Fleming’s Right Hand Rule,
to solve the rest of the question.
As we want the force acting on our charge to be parallel to the current direction,
D

we see that this will happen when the charge moves towards the wire9 .
Answer: (A)

10.30 Electron Configuration of a Potassium atom


We can alalyze and eliminate
A The n = 3 shell has unfilled d-subshells. So this is NOT TRUE.

B The 4s subshell only has one electron. The s subshell can ‘hold’ two electrons so this
is also NOT TRUE.

C Unknown.
9
Don’t forget to bring your right hand to the exam

©2009 David S. Latchman


82 GR8677 Exam Solutions
D The sum of all the electrons, we add all the superscripts, gives 19. As this is a
ground state, a lone potassium atom, we can tell that the atomic number is 19.
So this is NOT TRUE.

E Potassium has one outer electron, like Hydrogen. So it will also have a spherically
symmetrical charge distribution.

10.31 Photoelectric Effect I


We are given
|eV| = hυ − W (10.85)
We recall that V is the stopping potential, the voltage needed to bring the current to
zero. As electrons are negatively charged, we expect this voltage to be negative.

FT
Answer: (A)

10.32 Photoelectric Effect II


Some history needs to be known here. The photoelectric effect was one of the exper-
iments that proved that light was absorbed in discreet packets of energy. This is the
experimental evidence that won Einstein the Nobel Prize in 1921.
RA
Answer: (D)

10.33 Photoelectric Effect III


The quantity W is known as the work function of the metal. This is the energy that is
needed to just liberate an electron from its surface.
Answer: (D)
D

10.34 Potential Energy of a Body


We recall that
dU
F=− (10.86)
dx
Given that
U = kx4 (10.87)
The force on the body becomes

d 4
F = − kx
dx
= −4kx3 (10.88)

Answer: (B)

David S. Latchman ©2009


Hamiltonian of a Body 83
10.35 Hamiltonian of a Body
The Hamiltonian of a body is simply the sum of the potential and kinetic energies.
That is
H =T+V (10.89)
where T is the kinetic energy and V is the potential energy. Thus

1
H = mv2 + kx4 (10.90)
2
We can also express the kinetic energy in terms of momentum, p. So

p2
H= + kx4 (10.91)
2m

FT
Answer: (A)

10.36 Principle of Least Action


Hamilton’s Principle of Least Action10 states
Z
Φ=
 
T q(t), q̇(t) − V q(t) dt (10.92)
RA
T

where T is the kinetic energy and V is the potential energy. This becomes
Z t2
1 2
 
Φ= mv − kx4 dt (10.93)
t1 2

Answer: (A)

10.37 Tension in a Conical Pendulum


D

This is a simple case of resolving the horizontal and vertical components of forces. So
we have

T cos θ = mg (10.94)
T sin θ = mrω2 (10.95)

Squaring the above two equations and adding gives

T2 = m2 g2 + m2 r2 ω4 (10.96)

Leaving us with  
T = m g2 + r2 ω4 (10.97)
Answer: (E)

©2009 David S. Latchman


84 GR8677 Exam Solutions
Table 10.1: Truth Table for OR-gate

Input 1 Input 2 Output


0 0 0
0 1 1
1 0 1
1 1 1

10.38 Diode OR-gate


This is an OR gate and can be illustrated by the truth table below.
Answer: (A)

10.39

FT
Gain of an Amplifier vs. Angular Frequency
We are given that the amplifier has some sort of relationship where
G = Kωa (10.98)
falls outside of the amplifier bandwidth region. This is that ‘linear’ part of the graph
on the log-log graph. From the graph, we see that, G = 102 , for ω = 3 × 105 second-1 .
RA
Substituting, we get
 a
102 = K 3 × 105
h  i
∴ log(102 ) = a log K 3.5 × 105
⇒a≈2−5 (10.99)
We can roughly estimate by subtracting the indices. So our relationship is of the form
G = Kω−2 (10.100)
Answer: (E)
D

10.40 Counting Statistics



We recall from section 9.4 , that he standard deviation of a counting rate is σ = N,
where N is the number of counts. We have a count of N = 9934, so the standard
deviation is
√ √
σ = N = 9934

≈ 10000
= 100 (10.101)
Answer: (A)
10
Write something on this

David S. Latchman ©2009


Binding Energy per Nucleon 85
10.41 Binding Energy per Nucleon
More of a knowledge based question. Iron is the most stable of all the others.11
Answer: (C)

10.42 Scattering Cross Section


We are told the particle density of our scatterer is ρ = 1020 nuclei per cubic centimeter.
Given the thickness of our scatterer is ` = 0.1 cm, the cross sectional area is
N
ρ=
V
N
=
A`

FT
N
⇒A= (10.102)
ρ`
Now the probability of striking a proton is 1 in a million. So

10.43 Coupled Oscillators


There are two ways this system can oscillate, one mass on the end moves a lot and the
RA
other two move out of in the opposite directions but not as much or the centermass
can be stationary and the two masses on the end move out of phase with each other. In
the latter case, as there isn’t any energy transfer between the masses, the period would
be that of a single mass-spring system. The frequency of this would simply be
r
1 k
f = (10.103)
2π m
where k is the spring constant and m is the mass.
Answer: (B)
D

10.43.1 Calculating the modes of oscillation


In case you require a more rigorous approach, we can calculate the modes of oscillation.
The Lagrangian of the system is
L=T−V
1 h i 1 h i
= m ẋ21 + 2ẋ22 + ẋ23 − k (x2 − x2 )2 + (x3 − x2 )2 (10.104)
2 2
The equation of motion can be found from
d ∂L ∂L
!
= (10.105)
dt ∂ẋn ∂xn
11
Write up on Binding Energy

©2009 David S. Latchman


86 GR8677 Exam Solutions
The equations of motion are

mẍ1 = k (x2 − x1 ) (10.106)


2mẍ2 = kx1 − 2kx2 + kx3 (10.107)
mẍ3 = −k (x3 − x2 ) (10.108)

The solutions of the equations are

x1 = A cos(ωt) x2 = B cos(ωt) x3 = C cos(ωt)


(10.109)
ẍ1 = −ω2 x1 ẍ2 = −ω2 x2 ẍ3 = −ω2 x3

Solving this, we get


 
k − mω2 x1 − kx2 = 0 (10.110)
 
−kx1 + 2k − 2mω2 x2 − kx3 = 0

FT
(10.111)
 
−kx2 + k − mω2 x3 = 0 (10.112)

We can solve the modes of oscillation by solving



k − mω2 −k 0
−k 2k − 2mω2 −k = 0 (10.113)
0 −k k − mω2

RA
Finding the determinant results in
   2  h  i
k − mω 2 k − mω − k − k k k − mω2
2 2 2
(10.114)

Solving, we get √
k k 2k
ω= ; ± (10.115)
m m m
Substituting ω = k/m into the equations of motion, we get

x1 = −x3 (10.116)
D

x2 = 0 (10.117)

We see that the two masses on the ends move out of phase with each other and the
middle one is stationary.

10.44 Collision with a Rod


Momentum will be conserved, so we can say

mv = MV
mv
V= (10.118)
M
Answer: (A)

David S. Latchman ©2009


Compton Wavelength 87
10.45 Compton Wavelength
We recall from subsection 7.8.2, the Compton Equation from Equation 7.33
h
∆λ = λ0 − λ = (1 − cos θ) (10.119)
me c
Let θ = 90◦ , we get the Compton Wavelength
h
λc = = 2.427 × 10−12 m (10.120)
me c
Answer: (C)

10.46 Stefan-Boltzmann’s Equation

FT
We recall the Stefan-Boltzmann’s Equation, Equation 5.13

P(T) = σT4 (10.121)


At temperature, T1 ,
P1 = σT1 = 10 mW (10.122)
We are given T2 = 2T1 , so
P2 = σT24
RA
= σ (2T1 )4
= 16T24
= 16P1 = 160 mW (10.123)
Answer: (E)

10.47 Franck-Hertz Experiment


The Franck-Hertz Experiment as seen in subsection 7.9.3 deals with the manner in
D

which electrons of certain energies scatter or collide with Mercury atoms. At certain
energy levels, the Mercury atoms can ‘absorb’ the electrons energy and be excited and
this occurs in discreet steps.
Answer: (C)

10.48 Selection Rules for Electronic Transitions


We recall the selection rules for photon emission
∆` = ±1 Orbital angular momentum
∆m` = 0, ±1 Magnetic quantum number
∆ms = 0 Secondary spin quantum number,
∆ j = 0, ±1 Total angular momentum

©2009 David S. Latchman


88 GR8677 Exam Solutions
NOT FINISHED
Answer: (D)

10.49 The Hamilton Operator


The time-independent Schrödinger equation can be written

Ĥψ = Eψ (10.124)

We can determine the energy of a quantum particle by regarding the classical nonrel-
ativistic relationship as an equality of expectation values.
* 2+
p
hHi = + hVi (10.125)
2m

FT
We can solve this through the substition of a momentum operator

~ ∂
p→ (10.126)
i ∂x
Substituting this into Equation 10.125 gives us
Z +∞ "
~ ∂2
#
hHi = ψ −

ψ + V(x)ψ dx
RA
−∞ 2m ∂x2
Z +∞

= ψ∗ i~ ψdx (10.127)
−∞ ∂t
So we can get a Hamiltonian operator


H → i~ (10.128)
∂t

Answer: (B)
D

10.50 Hall Effect


The Hall Effect describes the production of a potential difference across a current
carrying conductor that has been placed in a magnetic field. The magnetic field is
directed perpendicularly to the electrical current.
As a charge carrier, an electron, moves through the conductor, the Lorentz Force
will cause a deviation in the carge carrier’s motion so that more charges accumulate
in one location than another. This asymmetric distribution of charges produces an
electric field that prevents the build up of more electrons. This ‘equilibrium’ voltage
across the conductor is known as the Hall Voltage and remains as long as a current
flows through our conductor.
As the deflection and hence, the Hall Voltage, is determined by the sign of the
carrier, this can be used to measure the sign of charge carriers.

David S. Latchman ©2009


Debye and Einstein Theories to Specific Heat 89
An equilibrium condition is reached when the electric force, generated by the accu-
mulated charge carriers, is equal the the magnetic force, that causes the accumulation
of charge carriers. Thus
Fm = evd B Fe = eE (10.129)
The current through the conductor is

I = nAvd e (10.130)

For a conductor of width, w and thickness, d, there is a Hall voltage across the width
of the conductor. Thus the electrical force becomes

Fe = eE
EVH
= (10.131)

FT
w
The magnetic force is
BI
Fm = (10.132)
neA
Equation 10.131 is equal to Equation 10.132, thus

eVH BI
=
RA
w newd
BI
∴ VH = (10.133)
ned
So for a measured magnetic field and current, the sign of the Hall voltage gives is the
sign of the charge carrier.
Answer: (C)

10.51 Debye and Einstein Theories to Specific Heat


D

The determination of the specific heat capacity was first deermined by the Law of
Dulong and Petite. This Law was based on Maxwell-Boltzmann statistics and was
accurate in its predictions except in the region of low temperatures. At that point there
is a departure from prediction and measurements and this is where the Einstein and
Debye models come into play.
Both the Einstein and Debye models begin with the assumption that a crystal is
made up of a lattice of connected quantum harmonic oscillators; choice B.
The Einstein model makes three assumptions

1. Each atom is a three-dimensional quantum harmonic oscillator.

2. Atoms do not interact with each other.

3. Atoms vibrate with the same frequency.

©2009 David S. Latchman


90 GR8677 Exam Solutions
Einstein assumed a quantum oscillator model, similar to that of the black body radi-
ation problem. But despite its success, his theory predicted an exponential decress in
heat capacity towards absolute zero whereas experiments followed a T3 relationship.
This was solved in the Debye Model.
The Debye Model looks at phonon contribution to specific heat capacity. This
theory correctly predicted the T3 proportionality at low temperatures but suffered at
intemediate temperatures.
Answer: (B)

10.52 Potential inside a Hollow Cube


By applying Gauss’ Law and drawing a Gaussian surface inside the cube, we see that
no charge is enclosed and hence no electric field12 . We can realte the electric field to

FT
the potential
E = −∇V (10.134)
Where V is the potential.
Gauss’ Law shows that with no enclosed charge we have no electric field inside our
cube. Thus
E = −∇V = 0 (10.135)
As Equation 10.134 is equal to zero, the potential is the same throughout the cube.13
RA
Answer: (E)

10.53 EM Radiation from Oscillating Charges


As the charge particle oscillates, the electric field oscillates as well. As the field oscillates
and changes, we would expect this changing field to affect a distant charge. If we
consider a charge along the xy-plane, looking directly along the x-axis, we won’t “see”
the charge oscillating but we would see it clearly if we look down the y-axis. If we
were to visualize the field, it would look like a doughnut around the x-axis. Based on
D

that analysis, we choose (C)


Answer: (C)

10.54 Polarization Charge Density


D = 0 E + P (10.136)

∇ · D = 0 ∇ · E + ∇ · P
D ∇ · E
= − σp
κ
12
Draw Cube at potential V with Gaussian Surface enclosing no charge
13
As we expect there to be no Electric Field, we must expect the potential to be the same throughout
the space of the cube. If there were differences, a charge place inside the cube would move.

David S. Latchman ©2009


Kinetic Energy of Electrons in Metals 91
Answer: (E)14

10.55 Kinetic Energy of Electrons in Metals


Electrons belong to a group known as fermions15 and as a result obey the Pauli Exclu-
sion Principle16 . So in the case of a metal, there are many fermions present each with
a different set of quantum numbers. The electron with the highest energy state is has
an energy value known as the Fermi Energy.
NOT FINSIHED
Answer: (B)

10.56 Expectation or Mean Value

FT
This is a definition question. The question states that for an operator Q,
Z +∞
hQi = ψ∗ Qψdx (10.137)
−∞

This is the very definition of the expectation or mean value of Q.


Answer: (C)
RA
10.57 Eigenfuction of Wavefunction
We are given the momentum operator as


p = −i~ (10.138)
∂x
With an eigenvalue of ~k. We can do this by trying each solution and seeing if they
match17
∂ψ
= ~kψ
D

− i~ (10.139)
∂x
A: ψ = cos kx We expect ψ, to have the form of an exponential function. Substituting
this into the eigenfuntion, Equation 10.139, we have

−i~ cos kx = −i~ (−k sin kx)
∂x
= i~k sin kx , ~kψ
ψ does not surive our differentiation and so we can eliminate it.
14
Check Polarization in Griffiths
15
Examples of fermions include electrons, protons and neutrons
16
The Pauli Exclusion Principle states that no two fermions may occupy the same quantum state
17
We can eliminate choices (A) & (B) as we would expect the answer to be an exponential function in
this case. These choices were just done for illustrative purposes and you should know to avoid them in
the exam.

©2009 David S. Latchman


92 GR8677 Exam Solutions
B: ψ = sin kx This is a similar case to the one above and we can eliminate for this
reason.

−i~ sin kx = −i~ (k cos kx)
∂x
= −i~k cos kx , ~kψ

Again we see that ψ does not survive when we apply our operator and so we can
eliminate this choice as well.

C: ψ = exp −ikx Substituting this into Equation 10.139, gives

∂ −ikx  
−i~ e = −i~ −ike−ikx
∂x
= −~ke−ikx , ~kψ

FT
Close but we are off, so we can eliminate this choice as well.

D: ψ = exp ikx If the above choice didn’t work, this might be more likely to.

∂ ikx  
−i~ e = −i~ ikeikx
∂x
= ~ke−ikx = ~kψ
RA
Success, this is our answer.

E: = ψ = exp −kx

∂ −kx  
−i~ e = −i~ −ke−kx
∂x
= −i~ke−kx , ~kψ

Again this choice does not work, so we can eliminate this as well

Answer: (D)
D

10.58 Holograms
The hologram is an image that produces a 3-dimensional image using both the Am-
plitude and Phase of a wave. Coherent, monochromatic light, such as from a laser, is
split into two beams. The object we wish to “photograph” is placed in the path of the
illumination beam and the scattered light falls on the recording medium. The second
beam, the reference beam is reflected unimpeded to the recording medium and these
two beams produces an interference pattern.
The intensity of light recorded on our medium is the same as the scattered light from
our object. The interference pattern is a result of phase changes as light is scattered off
our object. Thus choices (I) and (II) are true.
Answer: (B)

David S. Latchman ©2009


Group Velocity of a Wave 93
10.59 Group Velocity of a Wave
We are given the dispersion relationship of a wave as
  12
ω2 = c2 k2 + m2 (10.140)
The Group Velocity of a Wave is

vg = (10.141)
dk
By differentiating Equation 10.140 with respect to k, we can determine th group velocity
2ωdω = 2c2 kdk
dω c2 k
⇒ =
dk ω

FT
c2 k
= √ (10.142)
c2 k2 + m2
We want to examine the cases as k → 0 and k → ∞.
As k → 0, we have
dω c2 0
= √
dk 0 + m2
=0 (10.143)
RA
As k → ∞, c2 k2 >> m2 the denominator becomes

c2 k2 + m ≈ c2 k2 (10.144)
Replacing the denominator for our group velocity gives
dω c2 k
= =c (10.145)
dk ck
Answer: (E)
D

10.60 Potential Energy and Simple Harmonic Motion


We are given a potential energy of
V(x) = a + bx2 (10.146)
We can determine the mass’s spring constant, k, from V 00 (x)

V 00 (x) = 2b = k (10.147)
The angular frequency, ω, is
k 2b
ω2 = = (10.148)
m m
We see this is dependent on b and m.
Answer: (C)

©2009 David S. Latchman


94 GR8677 Exam Solutions
10.61 Rocket Equation I
We recall from the rocket equation that u in this case is the speed of the exaust gas
relative to the rocket.
Answer: (E)

10.62 Rocket Equation II


The rocket equation is
dv dm
m +u =0 (10.149)
dt dt
Solving this equation becomes

mdv = udm

FT
Z v Z m
dm
dv = u
0 m0 m
m
 
v = u ln (10.150)
m0
This fits none of the answers given.
Answer: (E)
RA
10.63 Surface Charge Density
This question was solved as ‘The Classic Image Problem’[2]. Below is an alternative
method but the principles are the same. Instead of determining the electrical potential,
as was done by Griffiths, we will find the electrical field of a dipole and determine the
surface charge density using
σ
E= (10.151)
0
D

Our point charge, −q will induce a +q on the grounded conducting plane. The
resulting electrical field will be due to a combination of the real charge and the ‘virtual’
induced charge. Thus

E = −E y ĵ = (E− + E+ ) ĵ
= 2E− ĵ (10.152)

Remember the two charges are the same, so at any point along the x-axis, or rather our
grounded conductor, the electrical field contributions from both charges will be the
same. Thus
q d
E− = cos θ where cos θ =
4πr2 r
qd
= (10.153)
4π0 r3

David S. Latchman ©2009


Maximum Power Theorem 95
Our total field becomes
2qd
E= (10.154)
4π0 r3

You may recognize that 2qd is the electrical dipole moment. Now, putting Equa-
tion 10.154 equal to Equation 10.151 gives us

σ qd
= (10.155)
0 2π0 r3

where r = D, we get
qd
σ= (10.156)
2πD2

FT
Answer: (C)

10.64 Maximum Power Theorem


We are given the impedance of our generator
RA
Z g = R g + jX g (10.157)

For the maximum power to be transmitted, the maximum power theorem states that the
load impedance must be equal to the complex conjugate of the generator’s impedance.

Z g = Z∗` (10.158)

Thus
D

Z` = R g + jX`
= R g − jX g (10.159)

Answer: (C)

10.65 Magnetic Field far away from a Current carrying


Loop
The Biot-Savart Law is
µ0 i d` × r̂
dB = (10.160)
4π r3

©2009 David S. Latchman


96 GR8677 Exam Solutions
Let θ be the angle between the radius, b and the radius vector, r, we get

µ0 i rd` cos θ b
B= 3
where cos θ =
4π r r
mu0 i d` cos θ
=
4π r2
µ0 i bd` √
= where r = b2 + h2
4π r3
µ0 i bd`
= where d` = b · dθ
4π (b2 + h2 ) 32
Z2π
µ0i b2
= · dθ
4π (b2 + h2 ) 23
0

FT
µ0 i b 2
= (10.161)
2 (b2 + h2 ) 32

we see that
B ∝ ib2 (10.162)

Answer: (B)
RA
10.66 Maxwell’s Relations
To derive the Maxwell’s Relations we begin with the thermodynamic potentials

First Law
dU = TdS − PdV (10.163)

Entalpy
D

H = E + PV
∴ dH = TdS + VdP (10.164)

Helmholtz Free Energy

F = E − TS
∴ dF = −SdT − PdV (10.165)

Gibbs Free Energy

G = E − TS + PV
∴ dG = −SdT + VdP (10.166)

David S. Latchman ©2009


Partition Functions 97
All of these differentials are of the form
∂z ∂z
! !
dz = dx + dy
∂x y
∂y x

= Mdx + Ndy

For the variables listed, we choose Equation 10.163 and applying the above condition
we get
∂U ∂U
! !
T= P= (10.167)
∂S V ∂V S
Thus taking the inverse of T, gives us

∂S
!
1
= (10.168)
∂U

FT
T V

Answer: (E)

10.67 Partition Functions


NOT FINISHED
RA
10.68 Particle moving at Light Speed
Answer: (A)

10.69 Car and Garage I


We are given the car’s length in its rest frame to be L0 = 5 meters and its Lorentz
Contracted length to be L = 3 meters. We can determine the speed from Equation 8.3
D

r
v2
L = L0 1 − 2
c
 2
3 v2
=1− 2
5 c
4
⇒v= c (10.169)
5
Answer: (C)

10.70 Car and Garage II


As the car approaches the garage, the driver will notice that things around him, in-
cluding the garage, are length contracted. We have calculated that the speed that

©2009 David S. Latchman


98 GR8677 Exam Solutions
he is travelling at to be, v = 0.8c, in the previous section. We again use the Length
Contraction formula, Equation 8.3, to solve this question.
!
v2
Lg = Lg 1 − 2
0
c
 
= 4 1 − 0.82
= 2.4 meters (10.170)

Answer: (A)

10.71 Car and Garage III


This is more of a conceptual question. What happens depends on whose frame of

FT
reference you’re in.
Answer: (E)

10.72 Refrective Index of Rock Salt and X-rays


No special knowledge is needed but a little knowledge always helps. You can start by
eliminating choices when in doubt.
RA
Choice A NOT TRUE Relativity says nothing about whether light is in a vacuum or
not. If anything, this choice goes against the postulates of Special Relativity. The
laws of Physics don’t change in vacuum.

Choice B NOT TRUE. X-rays can “transmit” signals or energy; any waveform can
once it is not distorted too much during propagation.

Choice C NOT TRUE. Photons have zero rest mass. Though the tachyon, a hypothet-
ical particle, has imaginary mass. This allows it to travel faster than the speed or
light though they don’t violate the principles of causality.
D

Choice D NOT TRUE. How or when we discover physical theories has no bearing
on observed properties or behavior; though according to some it may seem so at
times18

Choice E The phase and group speeds can be different. The phase velocity is the rate at
which the crests of the wave propagate or the rate at which the phase of the wave
is moving. The group speed is the rate at which the envelope of the waveform
18
There is a quote by Douglas Adams[3],
There is a theory which states that is ever anyone discovers exactly what the Universe is
for and why it is here, it will instantly disappear and be replaced by something even more
bizarre and inexplicable.
There is another which states this has already happened.
So maybe the order in which discoveries are made matters. Who am I to question Douglas Adams?

David S. Latchman ©2009


Thin Flim Non-Reflective Coatings 99
is moving or rather it’s the rate at which the amplitude varies in the waveform.
We can use this principle of n < 1 materials to create X-ray mirrors using “total
external reflection”.
Answer: (E)

10.73 Thin Flim Non-Reflective Coatings


To analyze this system, we consider our lens with refractive index, n3 , being coated by
our non-reflective coating of refractive index, n2 , and thickness, t, in air with refractive
index, n1 , where
n1 < n2 < n3 (10.171)
As our ray of light in air strikes the first boundary, the coating, it moves from a less

FT
optically dense medium to a more optically dense one. At the point where it reflects,
there will be a phase change in the reflected wave. The transmitted wave goes through
without a phase change.
The refracted ray passes through our coating to strike our glass lens, which is
optically more dense than our coating. As a result there will be a phase change in
our reflected ray. Destructive interference occurs when the optical path difference, 2t,
occurs in half-wavelengths multiples. So
1 λ
 
RA
2t = m + (10.172)
2 n2
where m = 0; 1; 2; 3. The thinnest possible coating occurs at m = 0. Thus

t= (10.173)
4 n2
We need a non-reflective coating that has an optical thicknes of a quarter wavelength.
Answer: (A)
D

10.74 Law of Malus


The Law of Malus states that when a perfect polarizer is placed in a polarized beam
of light, the intensity I, is given by

I = I0 cos2 θ (10.174)

where θ is the angle between the light’s plane of polarization and the axis of the
polarizer. A beam of light can be considered to be a uniform mix of plane polarization
angles and the average of this is
Z 2π
I = I0 cos2 θ
0
1
= I0 (10.175)
2

©2009 David S. Latchman


100 GR8677 Exam Solutions
So the maximum fraction of transmitted power through all three polarizers becomes
 3
1 I0
I3 = = (10.176)
2 8
Answer: (B)

10.75 Geosynchronous Satellite Orbit


We can relate the period or the angluar velocity of a satellite and Newton’s Law of
Gravitation  2
2π GMm
mRω = mR
2
= (10.177)
T R2
where M is the mass of the Earth, m is the satellite mass and RE is the orbital radius.

FT
From this we can get a relationship between the radius of orbit and its period, which
you may recognize as Kepler’s Law.
R3 ∝ T2 (10.178)
We can say
R3E ∝ (80)2 (10.179)
R3S ∝ (24 × 60) 2
(10.180)
RA
(10.181)
Dividing Equation 10.180 and Equation 10.181, gives

24 × 60 2
 3 
RS

=
RE 80
RS = 18 RE
3 2 3
(10.182)
Answer: (B)
D

10.76 Hoop Rolling down and Inclined Plane


As the hoop rolls down the inclined plane, its gravitational potential energy is con-
verted to translational kinetic energy and rotational kinetic energy
1 1
Mgh = Mv2 + Iω2 (10.183)
2 2
Recall that v = ωR, Equation 10.183 becomes
1 1 
MgH = MR2 ω2 + MR2 ω2 (10.184)
2 2
Solving for ω leaves
! 12
gh
ω= 2 (10.185)
R

David S. Latchman ©2009


Simple Harmonic Motion 101
The angular momentum is
L = Iω (10.186)
Substituting Equation 10.185 gives us
!1
gh 2
L = MR 2
R
p
= MR gh (10.187)

Answer: (A)

10.77 Simple Harmonic Motion

FT
We are told that a particle obeys Hooke’s Law, where

F = −kx (10.188)

We can write the equation of motion as

k
mẍ − kx where ω2 =
m
RA
where
 
x = A sin ωt + φ (10.189)
 
and ẋ = ωA cos ωt + φ (10.190)

We are told that


1  
= sin ωt + φ (10.191)
2
We can show that √
  3
cos ωt + φ = (10.192)
D

2
Substituting this into Equation 10.190 gives

3
ẋ = 2π f A ·
√ 2
= 3π f A (10.193)

Amswer: (B)

10.78 Total Energy between Two Charges


We are told three things

1. There is a zero potential energy, and

©2009 David S. Latchman


102 GR8677 Exam Solutions
2. one particle has non-zero speed and hence kinetic energy.

3. No radiation is emitted, so no energy is lost.


The total energy of the system is

E = Potential Energy + Kinetic Energy


= 0 + (KE > 0)
>0 (10.194)

Applying the three condition, we expect the total energy to be positive and constant.
Answer: (C)

10.79 Maxwell’s Equations and Magnetic Monopoles

FT
You may have heard several things about the ∇·B = 0 equation in Maxwell’s Laws. One
of them is there being no magnetic monopoles or charges. There are some implications
to this. No charge implies that the amount of field lines that enter a Gaussian surface
must be equal to the amount of field lines that leave. So using this principle we know
from the electric form of this law we can get an answer to this question.
Choice A The number of field lines that enter is the same as the number that leaves.
So this does not violate the above law.
RA
Choice B Again we see that the number of field lines entering is the same as the
number leaving.

Choice C The same as above

Choice D In this case, we see that the field lines at the edge of the Gaussian Surface
are all leaving; no field lines enter the surface. This is also what we’d expect the
field to look like for a region bounded by a magnetic monopole.

Choice E The field loops in on itself, so the total number of field lines is zero. This fits
D

with the above law.


Answer: (D)

10.80 Gauss’ Law


To determine an electric field that could exist in a region of space with no charges we
turn to Gauss’ Law.
∇·E=0 (10.195)
or rather
∂ ∂ ∂
Ex + E y + Ez = 0 (10.196)
∂x ∂y ∂z
So we analyze each choice in turn to get our answer.

David S. Latchman ©2009


Biot-Savart Law 103
Choice A

E = 2xyî − xyk̂
∂ ∂
∇·E= 2xy + (−xz)
∂x ∂z
= 2y + x , 0 (10.197)

Choice B

E = −xy jˆ + xzk̂
∂ ∂
∇·E= (−xy) + xz
∂y ∂z
= −x + x = 0 (10.198)

Choice C

∇·E=
FT
E = xzî + xz jˆ

∂x
=z+0,0
xz +
∂y

xz

(10.199)
RA
Choice D

ˆ
E = xyz(î + j)
∂ ∂
∇·E= xyz + xyz
∂x ∂y
= yz + xz , 0 (10.200)

Choice E
D

E = xyzî

∇·E= xyz
∂x
= yz , 0 (10.201)

Answer: (B)

10.81 Biot-Savart Law


We can determine the magnetic field produced by our outer wire from the Biot-Savart
Law
µ0 d` × r
dB = (10.202)
4π r3

©2009 David S. Latchman


104 GR8677 Exam Solutions
As our radius and differential length vectors are orthogonal, the magnetic field works
out to be
µ0 d`r
dB = I
4π r3
µ0 I rdθ
= ·
4π r2
µ0 I
Z 2π
B= dθ
4πr 0
µ0 I
= (10.203)
2b
We know from Faraday’s Law, a changing magnetic flux induces a EMF,

FT

E = (10.204)
dt
where Φ = BA. The magnetic flux becomes

µ0 I
Φ= · πa2 (10.205)
2b
The induced EMF becomes
RA
µ0 π
!
a2 dI
E =
2 b dt
µ0 π
!
a2
= ωI0 sin ωt (10.206)
2 b

Answer: (B)

10.82 Zeeman Effect and the emission spectrum of atomic


D

gases
Another knowledge based question best answered by the process of elimination.

Stern-Gerlach Experiemnt The Stern-Gerlach Experiment has nothing to do with


spectral emissions. This experiment, performed by O. Stern and W. Gerlach
in 1922 studies the behavior of a beam atoms being split in two as they pass
through a non-uniform magnetic field.

Stark Effect The Stark Effect deals with the shift in spectral lines in the presence of
electrical fields; not in magnetic fields.

Nuclear Magnetic Moments of atoms Close, the splitting seen in the Stern-Gerlach
Experiment is due to this. Emission spectrum typically deals with electrons and
so we would expect it to deal with electrons on some level.

David S. Latchman ©2009


Spectral Lines in High Density and Low Density Gases 105
Emission lines are split in two Closer but still not accurate. There is splitting but in
some cases it may be more than two.

Emission lines are greater or equal than in the absence of the magnetic field This we
know to be true.

The difference in the emission spectrum of a gas in a magnetic field is due to the
Zeeman effect.
Answer: (E)

10.83 Spectral Lines in High Density and Low Density


Gases

FT
We expect the spectral lines to be broader in a high density gas and narrower in a low
density gas ue to the increased colissions between the molecules. Atomic collisions
add another mechanism to transfer energy.[4]
Answer: (C)

10.84 Term Symbols & Spectroscopic Notation


RA
To determine the term symbol for the sodium ground state, we start with the electronic
configuration. This is easy as they have given us the number of electrons the element
has thus allowing us to fill sub-shells using the Pauli Exclusion Principle. We get

1s2 , 2s2 , 2p6 , 3s1 (10.207)

We are most interested in the 3s1 sub-shell and can ignore the rest of the filled sub-
shells. As we only have one valence electron then ms = +1/2. Now we can calculate
the total spin quantum number, S. As there is only one unpaired electron,
D

1
S= (10.208)
2
Now we can calculate the total angular momentum quantum number, J = L + S. As
the 3s sub-shell is half filled then
L=0 (10.209)
This gives us
1
J= (10.210)
2
and as L = 0 then we use the symbol S. Thus our term equation becomes
2
S 12 (10.211)

Answer: (B)

©2009 David S. Latchman


106 GR8677 Exam Solutions
10.85 Photon Interaction Cross Sections for Pb
Check Brehm p. 789
Answer: (B)

10.86 The Ice Pail Experiment


Gauss’ law is equivalent to Coulomb’s Law because Coulomb’s Law is an inverse
square law; testing one is a valid test of the other. Much of our knowledge of the
consequences of the inverse square law came from the study of gravity. Jason Priestly
knew that there is no gravitational field within a spherically symmetrical mass distri-
bution. It was suspected that was the same reason why a charged cork ball inside a
charged metallic container isn’t attracted to the walls of a container.

FT
Answer: (E)

10.87 Equipartition of Energy and Diatomic Molecules


To answer this question, we will turn to the equipartition of energy equation
!
f
cv = R (10.212)
RA
2

where f is the number of degrees of freedom. In the case of Model I, we see that So the

Table 10.2: Specific Heat, cv for a diatomic molecule

Degrees of Freedom Model I Model II


Translational 3 3
Rotational 2 2
Vibrational 0 2
D

Total 5 7

specific heats for Models I & II are


5 7
cvI = Nk cvII = Nk
2 2
Now we can go about choosing our answer
Choice A From our above calculations, we see that cvI = 5/2Nk. So this choice is
WRONG.

Choice B Again, our calculations show that the specific heat for Model II is larger than
than of Model I. This is due to the added degrees of freedom (vibrational) that it
possesses. So this choice is WRONG.

David S. Latchman ©2009


Fermion and Boson Pressure 107
C & D They both contradict the other and they both contradict Choice (E).

E This is TRUE. We know that at higher temperatures we have an additional degree


of freedom between our diatomic molecule.
Answer: (E)

10.88 Fermion and Boson Pressure


To answer this question, we must understand the differences between fermions and
bosons. Fermions follow Fermi-Dirac statistics and their behavior is obey the Pauli
Exclusion Principle. Basically, this states that no two fermions may have the same
quantum state. Bosons on the other hand follow Bose-Einstein statistics and several
bosons can occupy the same quantum state.

FT
As the temperature of a gas drops, the particles are going to fill up the available
energy states. In the case of fermions, as no two fermions can occupy the same state,
then these particles will try to occupy all the energy states it can until the highest is
filled. Bosons on the other hand can occupy the same state, so they will all ‘group’
together for the lowest they can. Classically, we don’t pay attention to this grouping,
so based on our analyis, we expect,

PF > PC > PB (10.213)


RA
where PB is the boson pressure, PC is the pressure with no quantum effects taking place
and PF to be the fermion pressure.
Answer: (B)

10.89 Wavefunction of Two Identical Particles


We are given the wavefunction of two identical particles,

1 h i
ψ = √ ψα (x1 )ψβ (x2 ) + ψβ (x1 )ψα (x2 )
D

(10.214)
2

This is a symmetric function and satisfies the relation

ψαβ (x2 , x1 ) = ψαβ (x1 , x2 ) (10.215)

Symmetric functions obey Bose-Einstein statistics and are known as bosons[6, 7, 8].
Upon examination of our choices, we see that19
electrons fermion

positrons fermion

protons fermion
19
You could have easily played the ‘one of thes things is not like the other...’ game

©2009 David S. Latchman


108 GR8677 Exam Solutions
neutron fermion

deutrons Boson
Incidentally, a anti-symmetric function takes the form,

1 h i
ψ = √ ψα (x1 )ψβ (x2 ) − ψβ (x1 )ψα (x2 ) (10.216)
2

and satisfies the relation


ψαβ (x2 , x1 ) = −ψαβ (x1 , x2 ) (10.217)
These obey Fermi-Dirac Statistics and are known as fermions.
Answer: (E)

FT
10.90 Energy Eigenstates
We may recognize this wavefunction from studying the particle in an infinite well
problem and see this is the n = 2 wavefunction. We know that

En = n2 E0 (10.218)

We are given that E2 = 2 eV. So


RA
1
E0 = E2
n2
2
= eV
4
1
= eV (10.219)
2
Answer: (C)
D

10.91 Bragg’s Law


We recall Bragg’s Law
2d sin θ = nλ (10.220)
Plugging in what we know, we determine λ to be

λ = 2(3Å)(sin 30)
= 2(3Å)(0.5)
= 3Å (10.221)

We employ the de Broglie relationship between wavelength and momentum

h
p= (10.222)
λ

David S. Latchman ©2009


Selection Rules for Electronic Transitions 109
We get

h
mv =
λ
h
⇒v=

6.63 × 10−34
= (10.223)
(9.11 × 10−31 )(3 × 10( − 10))

We can determine the order of our answer by looking at the relevant indices

− 34 − (−31) − (−10) = 7 (10.224)

We see that (D) is close to what we are looking for.

FT
Answer: (D)

10.92 Selection Rules for Electronic Transitions


The selection rules for an electric dipole transition are[9]

∆` = ±1 Orbital angular momentum


RA
∆m` = 0, ±1 Magnetic quantum number
∆ms = 0 Secondary spin quantum number,
∆j = 0, ±1 Total angular momentum

We have no selection rules for spin, ∆s, so we can eliminate this choice.
Answer: (D)

10.93 Moving Belt Sander on a Rough Plane


D

We know the work done on a body by a force is

W =F×x (10.225)

We can relate this to the power of the sander; power is the rate at which work is done.
So
dW
P=
dt
dx
= F = Fv (10.226)
dt
The power of the sander can be calculated

P = VI (10.227)

©2009 David S. Latchman


110 GR8677 Exam Solutions
where V and I are the voltage across and the current through the sander. By equating
the Mechanical Power, Equation 10.226 and the Electrical Power, Equation 10.227, we
can determine the force that the motor exerts on the belt.
VI
F=
v
120 × 9
=
10
= 108 N (10.228)

The sander is motionless, so


F − µR = 0 (10.229)
where R is the normal force of the sander pushing against the wood. Thus the coefficient
of friction is

FT
F 108
µ= = = 1.08 (10.230)
R 100
Answer: (D)

10.94 RL Circuits
When the switch, S, is closed, a magnetic field builds up within the inductor and the
RA
inductor stores energy. The charging of the inductor can be derived from Kirchoff’s
Rules.
dI
E − IR − L = 0 (10.231)
dt
and the solution to this is
R1 t
  
I(t) = I0 1 − exp (10.232)
L
where the time constant, τ1 = L/R1 .
We can find the voltage across the resistor, R1 , by multiplying the above by R1 ,
D

giving us

R1 t
  
V(t) = R1 · I0 1 − exp
L
R1 t
  
= E 1 − exp (10.233)
L
The potential at A can be found by measuring the voltage across the inductor. Given
that

E − VR1 − VL = 0
∴ VL = E − VR1
R1 t
 
= E exp (10.234)
L

David S. Latchman ©2009


Carnot Cycles 111
This we know to be an exponential decay and (fortunately) limits our choices to either
(A) or (B)20
The story doesn’t end here. If the inductor was not present, the voltage would
quickly drop and level off to zero but with the inductor present, a change in current
means a change in magnetic flux; the inductor opposes this change. We would expect
to see a reversal in the potential at A. Since both (A) and (B) show this flip, we need to
think some more.
The energy stored by the inductor is

1 2 1 E 2
 
UL = LI0 = L (10.235)
2 2 R1

With S opened, the inductor is going to dump its energy across R2 and assuming that
the diode has negligible resistance, all of this energy goes to R2 . Thus

FT
!2
1 VR2
U= L (10.236)
2 R2
The above two equations are equal, thus
E VR2
=
R1 R2
VR2 = 3E (10.237)
RA
We expect the potential at A to be larger when S is opened. Graph (B) fits this choice.
Answer: B

10.95 Carnot Cycles


The Carnot Cycle is made up of two isothermal transformations, KL and MN, and two
adiabatic transformations, LM and NK. For isothermal transformations, we have

PV = nRT = a constant (10.238)


D

For adiabatic transformations, we have

PV γ = a constant (10.239)

where γ = CP /CV .
For the KL transformation, dU = 0.

Q2 = WK→L
Z VL
∴ WK→L = PdV
VK
VK
 
= nRT2 ln (10.240)
VL
20
If you get stuck beyond this point, you can guess. The odds are now in your favor.

©2009 David S. Latchman


112 GR8677 Exam Solutions
For the LM transformation,
γ γ
PL VL = PM VM (10.241)
For the MN transformation, dU = 0.
Q1 = WM→N
Z VN
∴ WM→N = PdV
VM
VN
 
= nRT1 ln (10.242)
VM
For the NK transformation,
γ γ
PN VN = PK VK (10.243)
Dividing Equation 10.241 and Equation 10.243, gives
γ γ

FT
PL VL PM VM
γ = γ
PK VK PN VN
VL VM
∴ = (10.244)
VK VN
The effeciency of an engine is defined
Q1
η=1− (10.245)
Q2
RA
We get
Q1 −WM→N
η=1− =1−
Q2 W
  K→L
VM
nRT1 ln VN
=1−  
nRT2 ln VK
VL
T1
=1− (10.246)
T2
1. We see that
D

Q1 T1
1− =1−
Q2 T2
Q1 T1
∴ = (10.247)
Q2 T2
Thus choice (A) is true.
2. Heat moves from the hot reservoir and is converted to work and heat. Thus
Q2 = Q1 + W (10.248)
The entropy change from the hot reservoir
dQ2
S= (10.249)
T
As the hot reservoir looses heat, the entropy decreases. Thus choice (B) is true.

David S. Latchman ©2009


First Order Perturbation Theory 113
3. For a reversible cycle, there is no net heat flow over the cycle. The change in
entropy is defined by Calusius’s Theorem.
I
dQ
=0 (10.250)
T

We see that the entropy of the system remains the same. Thus choice (C) is false.

4. The efficieny is defined


W
η= (10.251)
Q2
This becomes
Q1
η=1−
Q2

FT
Q2 − Q1
= (10.252)
Q2
Thus W = Q2 − Q1 . So choice (D) is true,

5. The effeciency is based on an ideal gas and has no relation to the substance used.
So choice (E) is also true.
Answer: (C)
RA
10.96 First Order Perturbation Theory
Perturbation Theory is a procedure for obtaining approximate solutions for a perturbed
state by studying the solutions of the unperturbed state[10]. We can, and shouldn’t,
calculate this in the exam.
We can get the first order correction to be ebergy eigenvalue[11]
0
E1n = hψ0n |H |ψ0n i (10.253)
D

From there we can get the first order correction to the wave function
X hψ0 |H0 |ψ0 i
m n
ψ1n = 0 0
 (10.254)
m,n
En − Em

and can be expressed as X


ψ1n = c(n)
m ψm
0
(10.255)
m,n

you may recognize this as a Fourier Series and this will help you knowing that the
perturbing potential is one period of a saw tooth wave. And you may recall that the
Fourier Series of a saw tooth wave form is made up of even harmonics.
Answer: (B)21
21
Griffiths gives a similar problem in his text[12]

©2009 David S. Latchman


114 GR8677 Exam Solutions
10.97 Colliding Discs and the Conservation of Angular
Momentum
As the disk moves, it possessed both angular and linear momentums. We can not
exactly add these two as they, though similar, are quite different beasts. But we can
define a linear angular motion with respect to some origin. As the two discs hit each
other, they fuse. This slows the oncoming disc. We can calculate the linear angular
momentum
L=r×p (10.256)
where p is the linear momentum and r is the distance from the point P to the center of
disc I. This becomes

Lv0 = MR × v0

FT
= −MRv0 (10.257)

It’s negative as the cross product of R and v0 is negative.


The Rotational Angular Momentum is

Lω0 = Iω0 (10.258)

Adding Equation 10.258 and Equation 10.257 gives the total angular momentum.
RA
L = Lω0 + Lv0
= Iω0 − MRv0
1 1
= MR2 ω0 − MR2 ω0
2 2
=0

Thus the total angular momentum at the point P is zero.


Answer: (A)
D

10.98 Electrical Potential of a Long Thin Rod


We have charge uniformly distributed along the glass rod. It’s linear charge density is

Q dQ
λ= = (10.259)
` dx
The Electric Potential is defined
q
V(x) = (10.260)
4π0 x
We can ‘slice’ our rod into infinitesimal slices and sum them to get the potential of the
rod.
1 λdx
dV = (10.261)
4π0 x

David S. Latchman ©2009


Ground State of a Positronium Atom 115
We assume that the potential at the end of the rod, x = ` is V = 0 and at some point
away from the rod, x, the potential is V. So
Z V Z x
λ dx
dV =
0 4π0 ` x
λ x
 
= ln (10.262)
4π0 `
Where x = 2`, Equation 10.262 becomes
Q 1 2`
 
V= ln
` 4π0 `
Q 1
= ln 2 (10.263)
` 4π0

FT
Answer: (D)

10.99 Ground State of a Positronium Atom


Positronium consists of an electron and a positron bound together to form an “exotic”
atom. As the masses of the electron and positron are the same, we must use a reduced-
mass correction factor to determine the enrgy levels of this system.22 . The reduced
RA
mass of the system is
1 1 1
= + (10.264)
µ me mp
Thus /mu is
me · mp
µ=
me + mp
me
= (10.265)
2
The ground state of the Hydrogen atom, in terms of the reduced mass is
D

µ
E1 = − E0
me
1
= − E0 (10.266)
2
where E0 = 13.6 eV.
Answer: (B)

10.100 The Pinhole Camera


A pinhole camera is simply a camera with no lens and a very small aperature. Light
passes through this hole to produce an inverted image on a screen. For the photography
22
Place cite here

©2009 David S. Latchman


116 GR8677 Exam Solutions
buffs among you, you know that by varying the size of a camera’s aperature can
accomplish various things; making the aperature bigger allows more light to enter and
produces a “brighter” picture while making the aperature smaller produces a sharper
image.
In the case of the pinhole camera, making the pinhole, or aperature, smaller pro-
duces a sharper image because it reduces “image overlap”. Think of a large hole as
a set of tiny pinholes places close to each other. This results in an infinite amount of
images overlapping each other and hence a blurry image. So to produce a sharp image,
it is best to use the smallest pinhole possible, the tradeoff being an image that’s not as
“bright”.
There are limits to the size of our pinhole. We can not say, for example, use an
infinitely small pinhole the produce the sharpest possible image. Beyond some point
diffraction effects take place and will ruin our image.
Consider a pinhole camera of length, D, with a pinhole of diameter, d. We know

FT
how much a beam of light will be diffracted through this pinhole by23
d sin θ = mλ (10.267)
this is the equation for the diffraction of a single slit. As θ is small and we will consider
first order diffraction effects, Equation 10.267 becomes
dθ = λ
λ
⇒θ= (10.268)
d
RA
The “size” of this spread out image is
y = 2θD
2λD
= (10.269)
d
So the ‘blur’ of our resulting image is
B= y−d
2λD
= −d (10.270)
d
D

We can see that we want to reduce y as much as possible. i.e. make it d. So Equa-
tion 10.270 becomes
2λD
0= −d
d
2λD
∴ =d
d √
Thus d = 2λD (10.271)
So we’d want a pinhole of that size to produce or sharpest image possible. This result
is close to the result that Lord Rayleigh used, which worked out to be

d = 1.9 Dλ (10.272)
Answer: (A)
23
Add image of pinhole camera

David S. Latchman ©2009


Appendix A

Constants & Important Equations

FT
A.1 Constants

Constant Symbol Value


Speed of light in a vacuum c 2.99 × 108 m/s
Gravitational Constant G 6.67 × 10−11 m3 /kg.s2
Rest Mass of the electron me 9.11 × 10−31 kg
RA
Avogadro’s Number NA 6.02 × 1023 mol-1
Universal Gas Constant R 8.31 J/mol.K
Boltzmann’s Constant k 1.38 × 10−23 J/K
Electron charge e 1.60 × 10−9 C
Permitivitty of Free Space 0 8.85 × 10−12 C2 /N.m2
Permeability of Free Space µ0 4π × 10−7 T.m/A
Athmospheric Pressure 1 atm 1.0 × 105 M/m2
Bohr Radius a0 0.529 × 10−10 m
D

A.2 Vector Identities

A.2.1 Triple Products

A · (B × C) = B · (C × A) = C · (A × B) (A.1)
A × (B × C) = B (A · C) − C (A · B) (A.2)
118 Constants & Important Equations
A.2.2 Product Rules

∇ f g = f ∇g + g ∇ f
  
(A.3)
∇ (A · B) = A × (∇ × B) + B × (∇ × A) + (A · ∇) B + (B · ∇) A (A.4)
∇ · f A = f (∇ · A) + A · ∇ f
 
(A.5)
∇ · (A × B) = B · (∇ × A) − A · (∇ × B) (A.6)
∇ × f A = f (∇ × A) − A × ∇ f
 
(A.7)
∇ × (A × B) = (B · ∇) A − (A · ∇) B + A (∇ · B) − B (∇ · A) (A.8)

A.2.3 Second Derivatives

∇ · (∇ × A) = 0 (A.9)
∇ × ∇f = 0

FT

(A.10)
∇ × (∇ × A) = ∇ (∇ · A) − ∇2 A (A.11)

A.3 Commutators
A.3.1 Lie-algebra Relations
RA
[A, A] = 0 (A.12)
[A, B] = −[B, A] (A.13)
[A, [B, C]] + [B, [C, A]] + [C, [A, B]] = 0 (A.14)

A.3.2 Canonical Commutator


[x, p] = i~ (A.15)

A.3.3 Kronecker Delta Function


D

(
0 if m , n;
δmn =
1 if m = n;
For a wave function Z
ψm (x)∗ ψn (x)dx = δmn (A.16)

A.4 Linear Algebra


A.4.1 Vectors
Vector Addition
The sum of two vectors is another vector
|αi + |βi = |γi (A.17)

David S. Latchman ©2009


Linear Algebra 119
Commutative
|αi + |βi = |βi + |αi (A.18)
Associative
|αi + |βi + |γi = |αi + |βi + |γi
 
(A.19)
Zero Vector
|αi + |0i = |αi (A.20)
Inverse Vector
|αi + | − αi = |0i (A.21)

FT
RA
D

©2009 David S. Latchman


120 Constants & Important Equations

FT
RA
D

David S. Latchman ©2009


Bibliography

[1] John J. Brehm and William J. Mullin. Introduction to the Structure of Matter, chapter
11-6, pages 567–571. Wiley, first edition, 1989.

[2] David J. Griffiths. Introduction to Electrodyanmics, chapter 3.2.1, pages 121–123.


Prentice Hall, third edition, 1999.

FT
[3] Douglas Adams. The restaurant at the end of the universe.

[4] Wikipedia. Spectral line — wikipedia, the free encyclopedia, 2009. [Online;
accessed 17-March-2009].

[5] Wikipedia. Term symbol — wikipedia, the free encyclopedia, 2008. [Online;
accessed 22-March-2009].
RA
[6] John J. Brehm and William J. Mullin. Introduction to the Structure of Matter, chapter
5-10, pages 283–287. Wiley, first edition, 1989.

[7] John J. Brehm and William J. Mullin. Introduction to the Structure of Matter, chapter
11-1, pages 539–540. Wiley, first edition, 1989.

[8] David J. Griffiths. Introduction to Quantum Mechanics, chapter 5.1.1, pages 203–205.
Prentice Hall, second edition, 2005.

[9] David J. Griffiths. Introduction to Quantum Mechanics, chapter 9.3.3, pages 359–362.
Prentice Hall, second edition, 2005.
D

[10] David J. Griffiths. Introduction to Quantum Mechanics, chapter 6.1.1, page 249.
Prentice Hall, second edition, 2005.

[11] David J. Griffiths. Introduction to Quantum Mechanics, chapter 6.1.2, pages 251–254.
Prentice Hall, second edition, 2005.

[12] David J. Griffiths. Introduction to Quantum Mechanics, chapter 6.1.2, page 254.
Prentice Hall, second edition, 2005.
Index

Amplifiers Current Density


GR8677 Q39, 84 GR8677 Q09, 71
Angular Momentum, see Rotational Mo-
tion Dielectrics
Archimedes’ Principle, 23 GR8677 Q03, 68
Digital Circuits
Bernoulli’s Equation, 24 GR8677 Q38, 84

FT
Binding Energy Doppler Effect, 19
GR8677 Q41, 85 Drag Force
Bohr Model GR8677 Q01, 67
GR8677 Q19, 76
Hydrogen Model, 49 Elastic Colissions
GR8677 Q05, 69
Celestial Mechanics, 21 Electricity
Circular Orbits, 22 GR8677 Q24, 78
RA
Elliptical Orbit, 23 Electron Spin
Escape Speed, 22 GR8677 Q27, 79
Hyperbolic Orbit, 23 Electronic Configuration
Kepler’s Laws, 22 GR8677 Q30, 81
Newton’s Law of Gravitation, 21 Elliptic Orbits, see Orbits
Orbits, 22 Energy
Parabolic Orbit, 23 Kinectic Energy, 15
Potential Energy, 22 Potential Energy, 15
Vis-viva Equation, 23 Work-Energy Theorem, 15
Center of Mass, see System of Particles Equation of Continuity, 24
D

Cetripetal Motion
Fleming’s Right Hand Rule
GR8677 Q06, 70
GR8677 Q29, 81
Circular Orbits, see Celestial Mechanics
Fluid Dynamics, 23
Commutators, 118
Archimedes’ Principle, 23
Canonical Commutators, 118
Bernoulli’s Equation, 24
Kronecker Delta Function, 118
Equation of Continuity, 24
Lie-algebra Relations, 118
Franck-Hertz Experiment, 55
Compton Effect, 52
GR8677 Q47, 87
Compton Wavelength
GR8677 Q45, 87 Gauss’ Law
Conductivity GR8677 Q10, 72
GR8677 Q23, 78 Gravitation, see Celestial Mechanics
Counting Statistics, 65
GR8677 Q40, 84 Hall Effect
Index 123
GR8677 Q50, 88 Parabolic Orbit, 23
Hamiltonian, 24 Oscillatory Motion, 16
GR8677 Q35, 83 Coupled Harmonic Oscillators, 17
Hooke’s Law, 15 GR8677 Q43, 85
Potential Energy of a Spring, 15 Damped Motion, 16
Hyperbolic Orbits, see Orbits Kinetic Energy, 16
Potential Energy, 16
Interference Simple Harmonic Motion Equation, 16
GR8677 Q13, 73 Small Oscillations, 17
Total Energy, 16
Kepler’s Laws, see Celestial Mechanics
Kinematics Parabolic Orbits, see Orbits
Circular Motion, 13 Parallel Axis Theorem, see Rotational Mo-
Linear Motion, 13 tion
Kronecker Delta Function, 118

FT
Particle Physics
Laboratory Methods Muon
GR8677 Q40, 84 GR8677 Q16, 75
Lagrangian, 24 Photoelectric Effect
Linear Algebra, 118 GR8677 Q31, 82
Vectors, 118 GR8677 Q32, 82
Lorentz Force Law GR8677 Q33, 82
GR8677 Q25, 78 Potential Energy, see Energy
RA
Lorentz Transformation GR8677 Q34, 82
GR8677 Q22, 77 Potential Energy of a Spring, see Hooke’s
Law
Maximum Power Theorem Principle of Least Action
GR8677 Q64, 95 GR8677 Q36, 83
Maxwell’s Laws Probability
GR8677 Q11, 73 GR8677 Q15, 74
Mechanics
GR8677 Q07, 70 Rolling Kinetic Energy, see Rotational Mo-
GR8677 Q08, 71 tion
Rotational Kinetic Energy, see Rotational
D

GR8677 Q37, 83
Moment of Inertia, see Rotational Motion Motion
Rotational Motion, 20
Newton’s Law of Gravitation, see Celestial Angular Momentum, 20
Mechanics Moment of Inertia, 20
Newton’s Laws, 14 Parallel Axis Theorem, 20
Impulse, 14 Rolling Kinetic Energy, 21
Momentum, 14 Rotational Kinetic Energy, 20
Nuclear Physics Torque, 20
Radioactive Decay
GR8677 Q17, 75 Satellite Orbits
GR8677 Q02, 68
Orbits Schrödinger’s Equation
Elliptical Orbit, 23 GR8677 Q18, 75
Hyperbolic Orbit, 23 Space-Time Interval

©2009 David S. Latchman


124 Index
GR8677 Q21, 77
Special Relativity
Doppler Shift
GR8677 Q12, 73
Energy
GR8677 Q20, 76
Specific Heat
GR8677 Q14, 73
Stefan-Boltzmann’s Equation, 40
GR8677 Q46, 87
System of Particles, 21
Center of Mass, 21

Thin Film Interference

FT
GR8677 Q73, 99
Torque, see Rotational Motion

Vector Identities, 117


Product Rules, 118
Second Derivatives, 118
Triple Products, 117
Vis-viva Equation, 23
RA
Wave Equation
GR8677 Q04, 68
Wave function
GR8677 Q28, 81
Work
Constant Force, 15
Work-Energy Theorem, see Energy

X-Rays
GR8677 Q26, 79
D

David S. Latchman ©2009

You might also like